Programs & Examples On #User manual

A user manual is also known as user guide. It is a technical communication document intended to give assistance to people using a particular system or software.

OS specific instructions in CMAKE: How to?

In General

You can detect and specify variables for several operating systems like that:

Detect Microsoft Windows

if(WIN32)
    # for Windows operating system in general
endif()

Or:

if(MSVC OR MSYS OR MINGW)
    # for detecting Windows compilers
endif()

Detect Apple MacOS

if(APPLE)
    # for MacOS X or iOS, watchOS, tvOS (since 3.10.3)
endif()

Detect Unix and Linux

if(UNIX AND NOT APPLE)
    # for Linux, BSD, Solaris, Minix
endif()

Your specific linker issue

To solve your issue with the Windows-specific wsock32 library, just remove it from other systems, like that:

if(WIN32)
    target_link_libraries(${PROJECT_NAME} bioutils wsock32)
else
    target_link_libraries(${PROJECT_NAME} bioutils)
endif()

Use of ~ (tilde) in R programming Language

The thing on the right of <- is a formula object. It is often used to denote a statistical model, where the thing on the left of the ~ is the response and the things on the right of the ~ are the explanatory variables. So in English you'd say something like "Species depends on Sepal Length, Sepal Width, Petal Length and Petal Width".

The myFormula <- part of that line stores the formula in an object called myFormula so you can use it in other parts of your R code.


Other common uses of formula objects in R

The lattice package uses them to specify the variables to plot.
The ggplot2 package uses them to specify panels for plotting.
The dplyr package uses them for non-standard evaulation.

How to convert BigInteger to String in java

String input = "0101";
BigInteger x = new BigInteger ( input , 2 );
String output = x.toString(2);

100% height minus header?

If your browser supports CSS3, try using the CSS element Calc()

height: calc(100% - 65px);

you might also want to adding browser compatibility options:

height: -o-calc(100% - 65px); /* opera */
height: -webkit-calc(100% - 65px); /* google, safari */
height: -moz-calc(100% - 65px); /* firefox */

also make sure you have spaces between values, see: https://stackoverflow.com/a/16291105/427622

Jquery- Get the value of first td in table

$(".hit").click(function(){
   var values = [];
   var table = $(this).closest("table");
   table.find("tr").each(function() {
      values.push($(this).find("td:first").html());
   });

   alert(values);    
});

You should avoid $(".hit") it's really inefficient. Try using event delegation instead.

List Git commits not pushed to the origin yet

how to determine if a commit with particular hash have been pushed to the origin already?

# list remote branches that contain $commit
git branch -r --contains $commit

How to JSON serialize sets?

I adapted Raymond Hettinger's solution to python 3.

Here is what has changed:

  • unicode disappeared
  • updated the call to the parents' default with super()
  • using base64 to serialize the bytes type into str (because it seems that bytes in python 3 can't be converted to JSON)
from decimal import Decimal
from base64 import b64encode, b64decode
from json import dumps, loads, JSONEncoder
import pickle

class PythonObjectEncoder(JSONEncoder):
    def default(self, obj):
        if isinstance(obj, (list, dict, str, int, float, bool, type(None))):
            return super().default(obj)
        return {'_python_object': b64encode(pickle.dumps(obj)).decode('utf-8')}

def as_python_object(dct):
    if '_python_object' in dct:
        return pickle.loads(b64decode(dct['_python_object'].encode('utf-8')))
    return dct

data = [1,2,3, set(['knights', 'who', 'say', 'ni']), {'key':'value'}, Decimal('3.14')]
j = dumps(data, cls=PythonObjectEncoder)
print(loads(j, object_hook=as_python_object))
# prints: [1, 2, 3, {'knights', 'who', 'say', 'ni'}, {'key': 'value'}, Decimal('3.14')]

How to draw a line in android

Another approach to draw a line programatically using ImageView

import android.app.Activity;
import android.graphics.Bitmap;
import android.graphics.Canvas;
import android.graphics.Color;
import android.graphics.Paint;
import android.graphics.Path;
import android.graphics.Typeface;
import android.os.Bundle;
import android.widget.ImageView;

public class Test extends Activity {
  ImageView drawingImageView;

  @Override
  public void onCreate(Bundle savedInstanceState) {
    super.onCreate(savedInstanceState);
    setContentView(R.layout.main);
    drawingImageView = (ImageView) this.findViewById(R.id.DrawingImageView);
    Bitmap bitmap = Bitmap.createBitmap((int) getWindowManager()
        .getDefaultDisplay().getWidth(), (int) getWindowManager()
        .getDefaultDisplay().getHeight(), Bitmap.Config.ARGB_8888);
    Canvas canvas = new Canvas(bitmap);
    drawingImageView.setImageBitmap(bitmap);

    // Line
    Paint paint = new Paint();
    paint.setColor(Color.GREEN);
    paint.setStrokeWidth(10);
    int startx = 50;
    int starty = 100;
    int endx = 150;
    int endy = 210;
    canvas.drawLine(startx, starty, endx, endy, paint);

  }
}

Change icons of checked and unchecked for Checkbox for Android

it's android:button="@drawable/selector_checkbox" to make it work

Win32Exception (0x80004005): The wait operation timed out

To all those who know more than me, rather than marking it unhelpful or misleading, read it one more time. I had issues with my Virtual Machine (VM) becoming unresponsive due to all resources being consumed by locked threads, so killing threads is the only option I had. I am not recommending this to anyone who are running long queries but may help to those who are stuck with unresponsive VM or something. Its up-to individuals to take the call. Yes it will kill your query but it saved my VM machine being destroyed.

Serverstack already answered similar question. It solved my issue with SQL on VM machine. Please check here

You need to run following command to fix issues with indexes.

exec sp_updatestats

What is a difference between unsigned int and signed int in C?

Because it's all just about memory, in the end all the numerical values are stored in binary.

A 32 bit unsigned integer can contain values from all binary 0s to all binary 1s.

When it comes to 32 bit signed integer, it means one of its bits (most significant) is a flag, which marks the value to be positive or negative.

Python assigning multiple variables to same value? list behavior

The code that does what I need could be this:

# test

aux=[[0 for n in range(3)] for i in range(4)]
print('aux:',aux)

# initialization

a,b,c,d=[[0 for n in range(3)] for i in range(4)]

# changing values

a[0]=1
d[2]=5
print('a:',a)
print('b:',b)
print('c:',c)
print('d:',d)

Result:

('aux:', [[0, 0, 0], [0, 0, 0], [0, 0, 0], [0, 0, 0]])
('a:', [1, 0, 0])
('b:', [0, 0, 0])
('c:', [0, 0, 0])
('d:', [0, 0, 5])

Aggregate / summarize multiple variables per group (e.g. sum, mean)

Interestingly, base R aggregate's data.frame method is not showcased here, above the formula interface is used, so for completeness:

aggregate(
  x = df1[c("x1", "x2")],
  by = df1[c("year", "month")],
  FUN = sum, na.rm = TRUE
)

More generic use of aggregate's data.frame method:

Since we are providing a

  • data.frame as x and
  • a list (data.frame is also a list) as by, this is very useful if we need to use it in a dynamic manner, e.g. using other columns to be aggregated and to aggregate by is very simple
  • also with custom-made aggregation functions

For example like so:

colsToAggregate <- c("x1")
aggregateBy <- c("year", "month")
dummyaggfun <- function(v, na.rm = TRUE) {
  c(sum = sum(v, na.rm = na.rm), mean = mean(v, na.rm = na.rm))
}

aggregate(df1[colsToAggregate], by = df1[aggregateBy], FUN = dummyaggfun)

How to delete all files older than 3 days when "Argument list too long"?

Another solution for the original question, esp. useful if you want to remove only SOME of the older files in a folder, would be smth like this:

find . -name "*.sess" -mtime +100 

and so on.. Quotes block shell wildcards, thus allowing you to "find" millions of files :)

How to merge two arrays in JavaScript and de-duplicate items

ES2019

You can use it like union(array1, array2, array3, ...)

/**
 * Merges two or more arrays keeping unique items. This method does
 * not change the existing arrays, but instead returns a new array.
 */
function union<T>(...arrays: T[]) {
  return [...new Set([...arrays].flat())];
}

It is ES2019 because of the flat() function, but you can use core-js to get it as a polyfill. The T here is TypeScript generic type, that you can remove if you are not using TypeScript. If you are using TypeScript, make sure to add "lib": ["es2019.array"] to compiler options in tsconfig.json.

or...

just use lodash _.union

What are the uses of "using" in C#?

using can be used to call IDisposable. It can also be used to alias types.

using (SqlConnection cnn = new SqlConnection()) { /*code*/}
using f1 = System.Windows.Forms.Form;

ERROR 1044 (42000): Access denied for 'root' With All Privileges

If you get an error 1044 (42000) when you try to run SQL commands in MySQL (which installed along XAMPP server) cmd prompt, then here's the solution:

  1. Close your MySQL command prompt.

  2. Open your cmd prompt (from Start menu -> run -> cmd) which will show: C:\Users\User>_

  3. Go to MySQL.exe by Typing the following commands:

C:\Users\User>cd\ C:\>cd xampp C:\xampp>cd mysql C:\xxampp\mysql>cd bin C:\xampp\mysql\bin>mysql -u root

  1. Now try creating a new database by typing:

    mysql> create database employee;
    

    if it shows:

    Query OK, 1 row affected (0.00 sec)
    mysql>
    

    Then congrats ! You are good to go...

pandas get column average/mean

You can use either of the two statements below:

numpy.mean(df['col_name'])
# or
df['col_name'].mean()

Bootstrap 3 navbar active li not changing background-color

You need to add CSS to .active instead of .active a.

Fiddle: http://jsfiddle.net/T5X6h/2/

Something like this:

.navbar-default .navbar-nav > .active{
    color: #000;
    background: #d65c14;
}
.navbar-default .navbar-nav > .active > a,
.navbar-default .navbar-nav > .active > a:hover,
.navbar-default .navbar-nav > .active > a:focus {
    color: #000;
    background: #d65c14;
}

g++ ld: symbol(s) not found for architecture x86_64

I had a similar warning/error/failure when I was simply trying to make an executable from two different object files (main.o and add.o). I was using the command:

gcc -o exec main.o add.o

But my program is a C++ program. Using the g++ compiler solved my issue:

g++ -o exec main.o add.o

I was always under the impression that gcc could figure these things out on its own. Apparently not. I hope this helps someone else searching for this error.

MVC pattern on Android

In my understanding, the way Android handles the MVC pattern is like:

You have an Activity, which serves as the controller. You have a class which responsibility is to get the data - the model, and then you have the View class which is the view.

When talking about the view most people think only for its visual part defined in the xml. Let's not forget that the View also has a program part with its constructors, methods and etc, defined in the java class.

How to convert Rows to Columns in Oracle?

If you are using Oracle 10g, you can use the DECODE function to pivot the rows into columns:

CREATE TABLE doc_tab (
  loan_number VARCHAR2(20),
  document_type VARCHAR2(20),
  document_id VARCHAR2(20)
);

INSERT INTO doc_tab VALUES('992452533663', 'Voters ID', 'XPD0355636');
INSERT INTO doc_tab VALUES('992452533663', 'Pan card', 'CHXPS5522D');
INSERT INTO doc_tab VALUES('992452533663', 'Drivers licence', 'DL-0420110141769');

COMMIT;

SELECT
    loan_number,
    MAX(DECODE(document_type, 'Voters ID', document_id)) AS voters_id,
    MAX(DECODE(document_type, 'Pan card', document_id)) AS pan_card,
    MAX(DECODE(document_type, 'Drivers licence', document_id)) AS drivers_licence
  FROM
    doc_tab
GROUP BY loan_number
ORDER BY loan_number;

Output:

LOAN_NUMBER   VOTERS_ID            PAN_CARD             DRIVERS_LICENCE    
------------- -------------------- -------------------- --------------------
992452533663  XPD0355636           CHXPS5522D           DL-0420110141769     

You can achieve the same using Oracle PIVOT clause, introduced in 11g:

SELECT *
  FROM doc_tab
PIVOT (
  MAX(document_id) FOR document_type IN ('Voters ID','Pan card','Drivers licence')
);

SQLFiddle example with both solutions: SQLFiddle example

Read more about pivoting here: Pivot In Oracle by Tim Hall

TypeError: 'list' object cannot be interpreted as an integer

For me i was getting this error because i needed to put the arrays in paratheses. The error is a bit tricky in this case...

ie. concatenate((a, b)) is right

not concatenate(a, b)

hope that helps.

reading text file with utf-8 encoding using java

I ran into the same problem every time it finds a special character marks it as ??. to solve this, I tried using the encoding: ISO-8859-1

BufferedReader br = new BufferedReader(new InputStreamReader(new FileInputStream("txtPath"),"ISO-8859-1"));

while ((line = br.readLine()) != null) {

}

I hope this can help anyone who sees this post.

What design patterns are used in Spring framework?

Factory pattern is also used for loading beans through BeanFactory and Application context.

Struct inheritance in C++

In C++, a structure's inheritance is the same as a class except the following differences:

When deriving a struct from a class/struct, the default access-specifier for a base class/struct is public. And when deriving a class, the default access specifier is private.

For example, program 1 fails with a compilation error and program 2 works fine.

// Program 1
#include <stdio.h>

class Base {
    public:
        int x;
};

class Derived : Base { }; // Is equivalent to class Derived : private Base {}

int main()
{
    Derived d;
    d.x = 20; // Compiler error because inheritance is private
    getchar();
    return 0;
}

// Program 2
#include <stdio.h>

struct Base {
    public:
        int x;
};

struct Derived : Base { }; // Is equivalent to struct Derived : public Base {}

int main()
{
    Derived d;
    d.x = 20; // Works fine because inheritance is public
    getchar();
    return 0;
}

How do you uninstall the package manager "pip", if installed from source?

I was using above command but it was not working. This command worked for me:

python -m pip uninstall pip setuptools

Easiest way to activate PHP and MySQL on Mac OS 10.6 (Snow Leopard), 10.7 (Lion), 10.8 (Mountain Lion)?

Open a good text editor (I'd recommend TextMate, but the free TextWrangler or vi or nano will do too), and open:

/etc/apache2/httpd.conf

Find the line:

"#LoadModule php5_module        libexec/apache2/libphp5.so"

And uncomment it (remove the #).

Download and install the latest MySQL version from mysql.com. Choose the x86_64 version for Intel (unless your Intel Mac is the original Macbook Pro or Macbook, which are not 64 bit chips. In those cases, use the 32 bit x86 version).

Install all the MySQL components. Using the pref pane, start MySQL.

In the Sharing System Pref, turn on (or if it was already on, turn off/on) Web Sharing.

You should now have Apache/PHP/MySQL running.

In 10.4 and 10.5 it was necessary to modify the php.ini file to point to the correct location of mysql.sock. There are reports that this is fixed in 10.6, but that doesn't appear to be the case for all of us, given some of the comments below.

Opening a .ipynb.txt File

Below is the easiest way in case if Anaconda is already installed.

1) Under "Files", there is an option called,"Upload".

2) Click on "Upload" button and it asks for the path of the file and select the file and click on upload button present beside the file.

Should a function have only one return statement?

Having a single exit point reduces Cyclomatic Complexity and therefore, in theory, reduces the probability that you will introduce bugs into your code when you change it. Practice however, tends to suggest that a more pragmatic approach is needed. I therefore tend to aim to have a single exit point, but allow my code to have several if that is more readable.

Changing plot scale by a factor in matplotlib

As you have noticed, xscale and yscale does not support a simple linear re-scaling (unfortunately). As an alternative to Hooked's answer, instead of messing with the data, you can trick the labels like so:

ticks = ticker.FuncFormatter(lambda x, pos: '{0:g}'.format(x*scale))
ax.xaxis.set_major_formatter(ticks)

A complete example showing both x and y scaling:

import numpy as np
import pylab as plt
import matplotlib.ticker as ticker

# Generate data
x = np.linspace(0, 1e-9)
y = 1e3*np.sin(2*np.pi*x/1e-9) # one period, 1k amplitude

# setup figures
fig = plt.figure()
ax1 = fig.add_subplot(121)
ax2 = fig.add_subplot(122)
# plot two identical plots
ax1.plot(x, y)
ax2.plot(x, y)

# Change only ax2
scale_x = 1e-9
scale_y = 1e3
ticks_x = ticker.FuncFormatter(lambda x, pos: '{0:g}'.format(x/scale_x))
ax2.xaxis.set_major_formatter(ticks_x)

ticks_y = ticker.FuncFormatter(lambda x, pos: '{0:g}'.format(x/scale_y))
ax2.yaxis.set_major_formatter(ticks_y)

ax1.set_xlabel("meters")
ax1.set_ylabel('volt')
ax2.set_xlabel("nanometers")
ax2.set_ylabel('kilovolt')

plt.show() 

And finally I have the credits for a picture:

Left: ax1 no scaling, right: ax2 y axis scaled to kilo and x axis scaled to nano

Note that, if you have text.usetex: true as I have, you may want to enclose the labels in $, like so: '${0:g}$'.

Moment.js - tomorrow, today and yesterday

I have similar solution, but allows to use locales:

    let date = moment(someDate);
    if (moment().diff(date, 'days') >= 1) {
        return date.fromNow(); // '2 days ago' etc.
    }
    return date.calendar().split(' ')[0]; // 'Today', 'yesterday', 'tomorrow'

WPF Label Foreground Color

The title "WPF Label Foreground Color" is very simple (exactly what I was looking for) but the OP's code is so cluttered it's easy to miss how simple it can be to set text foreground color on two different labels:

<StackPanel>
    <Label Foreground="Red">Red text</Label>
    <Label Foreground="Blue">Blue text</Label>
</StackPanel>

In summary, No, there was nothing wrong with your snippet.

How to make bootstrap 3 fluid layout without horizontal scrollbar

The only thing that assisted me was to set margin:0px on the topmost <div class="row"> in my html DOM.

This again wasn't the most appealing way to solve the issue, but as it is only in one place I put it inline.

As an fyi the container-fluid and apparent bootstrap fixes only introduced an increased whitespace on either side of the visible page... :( Although I came across my solution by reading through the back and forth on the github issue - so worthwhile reading.

Error - SqlDateTime overflow. Must be between 1/1/1753 12:00:00 AM and 12/31/9999 11:59:59 PM

DateTime.MinValue and DateTime.MaxValue

DateTime.MinValue = 1/1/0001 12:00:00 AM

DateTime.MaxValue = 23:59:59.9999999, December 31, 9999, 

                exactly one 100-nanosecond tick 

                before 00:00:00, January 1, 10000 

How to insert pandas dataframe via mysqldb into database?

This has worked for me. At first I've created only the database, no predefined table I created.

from platform import python_version
print(python_version())
3.7.3

path='glass.data'
df=pd.read_csv(path)
df.head()


!conda install sqlalchemy
!conda install pymysql

pd.__version__
    '0.24.2'

sqlalchemy.__version__
'1.3.20'

restarted the Kernel after installation.

from sqlalchemy import create_engine
engine = create_engine('mysql+pymysql://USER:PASSWORD@HOST:PORT/DATABASE_NAME', echo=False)

try:
df.to_sql(name='glasstable',con=engine,index=False, if_exists='replace')
print('Sucessfully written to Database!!!')

except Exception as e:
    print(e)

How do I check if a string contains another string in Objective-C?

Try this:

Swift 4.1 , 4.2:

let stringData = "Black board"

//swift quick way and case sensitive
if stringData.contains("bla") {
    print("data contains string");
}

//case sensitive
if stringData.range(of: "bla",options: .caseInsensitive) != nil {
    print("data contains string");
}else {
    print("data does not contains string");
}

For Objective-C:

NSString *stringData = @"Black board";

//Quick way and case sensitive
if ([stringData containsString:@"bla"]) {
    NSLog(@"data contains string");
}

//Case Insensitive
if ([stringData rangeOfString:@"bla" options:NSCaseInsensitiveSearch].location != NSNotFound) {
   NSLog(@"data contains string");
}else {
   NSLog(@"data does not contain string");
}

Android Facebook integration with invalid key hash

If you are using Google Play App signing:

Open the App signing section in Google Play Console, and get the SHA-1 hash under App signing certificate. Then convert it to Base64, for example with this tool: Hexadecimal -> base64 string decoder

Console screenshot

Convert to Base64 screenshot

Angular ngClass and click event for toggling class

Angular6 using the renderer2 without any variables and a clean template:

template:

<div (click)="toggleClass($event,'testClass')"></div>

in ts:

toggleClass(event: any, class: string) {
  const hasClass = event.target.classList.contains(class);

  if(hasClass) {
    this.renderer.removeClass(event.target, class);
  } else {
    this.renderer.addClass(event.target, class);
  }
}

One could put this in a directive too ;)

Should I use != or <> for not equal in T-SQL?

One alternative would be to use the NULLIF operator other than <> or != which returns NULL if the two arguments are equal NULLIF in Microsoft Docs. So I believe WHERE clause can be modified for <> and != as follows:

NULLIF(arg1, arg2) IS NOT NULL

As I found that, using <> and != doesn't work for date in some cases. Hence using the above expression does the needful.

Getting ssh to execute a command in the background on target machine

First follow this procedure:

Log in on A as user a and generate a pair of authentication keys. Do not enter a passphrase:

a@A:~> ssh-keygen -t rsa
Generating public/private rsa key pair.
Enter file in which to save the key (/home/a/.ssh/id_rsa): 
Created directory '/home/a/.ssh'.
Enter passphrase (empty for no passphrase): 
Enter same passphrase again: 
Your identification has been saved in /home/a/.ssh/id_rsa.
Your public key has been saved in /home/a/.ssh/id_rsa.pub.
The key fingerprint is:
3e:4f:05:79:3a:9f:96:7c:3b:ad:e9:58:37:bc:37:e4 a@A

Now use ssh to create a directory ~/.ssh as user b on B. (The directory may already exist, which is fine):

a@A:~> ssh b@B mkdir -p .ssh
b@B's password: 

Finally append a's new public key to b@B:.ssh/authorized_keys and enter b's password one last time:

a@A:~> cat .ssh/id_rsa.pub | ssh b@B 'cat >> .ssh/authorized_keys'
b@B's password: 

From now on you can log into B as b from A as a without password:

a@A:~> ssh b@B

then this will work without entering a password

ssh b@B "cd /some/directory; program-to-execute &"

Resolving MSB3247 - Found conflicts between different versions of the same dependent assembly

This warning generated for default ASP.NET MVC 4 beta see here

In, any cast this Warning can be eliminated by manually editing the .csproj file for your project.

modify........: Reference Include="System.Net.Http"

to read ......: Reference Include="System.Net.Http, Version=4.0.0.0"

How to make sure that a certain Port is not occupied by any other process

You can use "netstat" to check whether a port is available or not.

Use the netstat -anp | find "port number" command to find whether a port is occupied by an another process or not. If it is occupied by an another process, it will show the process id of that process.

You have to put : before port number to get the actual output

Ex netstat -anp | find ":8080"

How to check if a word is an English word with Python?

For a semantic web approach, you could run a sparql query against WordNet in RDF format. Basically just use urllib module to issue GET request and return results in JSON format, parse using python 'json' module. If it's not English word you'll get no results.

As another idea, you could query Wiktionary's API.

Curl and PHP - how can I pass a json through curl by PUT,POST,GET

I was Working with Elastic SQL plugin. Query is done with GET method using cURL as below:

curl -XGET http://localhost:9200/_sql/_explain -H 'Content-Type: application/json' \
-d 'SELECT city.keyword as city FROM routes group by city.keyword order by city'

I exposed a custom port at public server, doing a reverse proxy with Basic Auth set.

This code, works fine plus Basic Auth Header:

$host = 'http://myhost.com:9200';
$uri = "/_sql/_explain";
$auth = "john:doe";
$data = "SELECT city.keyword as city FROM routes group by city.keyword order by city";

function restCurl($host, $uri, $data = null, $auth = null, $method = 'DELETE'){
    $ch = curl_init();

    curl_setopt($ch, CURLOPT_URL, $host.$uri);
    curl_setopt($ch, CURLOPT_CUSTOMREQUEST, $method);
    curl_setopt($ch, CURLOPT_HTTPHEADER, array('Content-Type: application/json'));
    curl_setopt($ch, CURLOPT_RETURNTRANSFER, true);

    if ($method == 'POST')
        curl_setopt($ch, CURLOPT_POST, 1);
    if ($auth)
        curl_setopt($ch, CURLOPT_USERPWD, $auth);
    if (strlen($data) > 0)
        curl_setopt($ch, CURLOPT_POSTFIELDS,$data);

    $resp  = curl_exec($ch);
    if(!$resp){
        $resp = (json_encode(array(array("error" => curl_error($ch), "code" => curl_errno($ch)))));
    }   
    curl_close($ch);
    return $resp;
}

$resp = restCurl($host, $uri); //DELETE
$resp = restCurl($host, $uri, $data, $auth, 'GET'); //GET
$resp = restCurl($host, $uri, $data, $auth, 'POST'); //POST
$resp = restCurl($host, $uri, $data, $auth, 'PUT'); //PUT

How do I print output in new line in PL/SQL?

Most likely you need to use this trick:

dbms_output.put_line('Hi' || chr(10) || 
                     'good' || chr(10) || 
                     'morning' || chr(10) || 
                     'friends' || chr(10));

What is code coverage and how do YOU measure it?

Code coverage has been explained well in the previous answers. So this is more of an answer to the second part of the question.

We've used three tools to determine code coverage.

  1. JTest - a proprietary tool built over JUnit. (It generates unit tests as well.)
  2. Cobertura - an open source code coverage tool that can easily be coupled with JUnit tests to generate reports.
  3. Emma - another - this one we've used for a slightly different purpose than unit testing. It has been used to generate coverage reports when the web application is accessed by end-users. This coupled with web testing tools (example: Canoo) can give you very useful coverage reports which tell you how much code is covered during typical end user usage.

We use these tools to

  • Review that developers have written good unit tests
  • Ensure that all code is traversed during black-box testing

How to Convert double to int in C?

I suspect you don't actually have that problem - I suspect you've really got:

double a = callSomeFunction();
// Examine a in the debugger or via logging, and decide it's 3669.0

// Now cast
int b = (int) a;
// Now a is 3668

What makes me say that is that although it's true that many decimal values cannot be stored exactly in float or double, that doesn't hold for integers of this kind of magnitude. They can very easily be exactly represented in binary floating point form. (Very large integers can't always be exactly represented, but we're not dealing with a very large integer here.)

I strongly suspect that your double value is actually slightly less than 3669.0, but it's being displayed to you as 3669.0 by whatever diagnostic device you're using. The conversion to an integer value just performs truncation, not rounding - hence the issue.

Assuming your double type is an IEEE-754 64-bit type, the largest value which is less than 3669.0 is exactly

3668.99999999999954525264911353588104248046875

So if you're using any diagnostic approach where that value would be shown as 3669.0, then it's quite possible (probable, I'd say) that this is what's happening.

How do you set the width of an HTML Helper TextBox in ASP.NET MVC?

The original answer is no longer working as written:

<%=Html.TextBox("test", new { style="width:50px" })%> 

will get you a text box with "{ style="width:50px" }" as its text content.

To adjust this for the current release of MVC 1.0, use the following script (note the addition of the second parameter - I have mine starting as blank, adjust accordingly based on your needs):

<%=Html.TextBox("test", "", new { style="width:50px" })%> 

Making href (anchor tag) request POST instead of GET?

Using jQuery it is very simple assuming the URL you wish to post to is on the same server or has implemented CORS

$(function() {
  $("#employeeLink").on("click",function(e) {
    e.preventDefault(); // cancel the link itself
    $.post(this.href,function(data) {
      $("#someContainer").html(data);
    });
  });
});

If you insist on using frames which I strongly discourage, have a form and submit it with the link

<form action="employee.action" method="post" target="myFrame" id="myForm"></form>

and use (in plain JS)

 window.addEventListener("load",function() {
   document.getElementById("employeeLink").addEventListener("click",function(e) {
     e.preventDefault(); // cancel the link
     document.getElementById("myForm").submit(); // but make sure nothing has name or ID="submit"
   });
 });

Without a form we need to make one

 window.addEventListener("load",function() {
   document.getElementById("employeeLink").addEventListener("click",function(e) {
     e.preventDefault(); // cancel the actual link
     var myForm = document.createElement("form");
     myForm.action=this.href;// the href of the link
     myForm.target="myFrame";
     myForm.method="POST";
     myForm.submit();
   });
 });

Move cursor to end of file in vim

This is quicker. Just use this

:$

Datatable to html Table

I have seen some solutions here worth noting, as Omer Eldan posted. but here follows. ASP C#

using System.Data;
using System.Web.UI.HtmlControls;

public static Table DataTableToHTMLTable(DataTable dt, bool includeHeaders)
{
    Table tbl = new Table();
    TableRow tr = null;
    TableCell cell = null;

    int rows = dt.Rows.Count;
    int cols = dt.Columns.Count;

    if (includeHeaders)
    {
        TableHeaderRow htr = new TableHeaderRow();
        TableHeaderCell hcell = null;
        for (int i = 0; i < cols; i++)
        {
            hcell = new TableHeaderCell();
            hcell.Text = dt.Columns[i].ColumnName.ToString();
            htr.Cells.Add(hcell);
        }
        tbl.Rows.Add(htr);
    }

    for (int j = 0; j < rows; j++)
    {
        tr = new TableRow();
        for (int k = 0; k < cols; k++)
        {
            cell = new TableCell();
            cell.Text = dt.Rows[j][k].ToString();
            tr.Cells.Add(cell);
        }
        tbl.Rows.Add(tr);
    }
    return tbl;
}

why this solution? Because you can easily just add this to a panel ie:

panel.Controls.Add(DataTableToHTMLTable(dtExample,true));

Second question , why do you have one column datatables and not just array's? Are you sure that these DataTables are uniform, because if the data is jagged then it's no use. If You really have to join these DataTables, there is many examples of Linq operations, or just use (beware though of same name columns as this will conflict in both linq operations and this solution if not handled):

public DataTable joinUniformTable(DataTable dt1, DataTable dt2)
{
    int dt2ColsCount = dt2.Columns.Count;
    int dt1lRowsCount = dt1.Rows.Count;

    DataColumn column;
    for (int i = 0; i < dt2ColsCount; i++)
    {
        column = new DataColumn();
        string colName = dt2.Columns[i].ColumnName;
        System.Type colType = dt2.Columns[i].DataType;
        column.ColumnName = colName;
        column.DataType = colType;
        dt1.Columns.Add(column);

        for (int j = 0; j < dt1lRowsCount; j++)
        {
            dt1.Rows[j][colName] = dt2.Rows[j][colName];
        }
    }
    return dt1;
}

and your solution would look something like:

panel.Controls.Add(DataTableToHTMLTable(joinUniformTable(joinUniformTable(LivDT,BathDT),BedDT),true));

interpret the rest, and have fun.

How can I use a carriage return in a HTML tooltip?

The latest specification allows line feed character, so a simple line break inside the attribute or entity &#10; (note that characters # and ; are required) are OK.

Java - Find shortest path between 2 points in a distance weighted map

Maintain a list of nodes you can travel to, sorted by the distance from your start node. In the beginning only your start node will be in the list.

While you haven't reached your destination: Visit the node closest to the start node, this will be the first node in your sorted list. When you visit a node, add all its neighboring nodes to your list except the ones you have already visited. Repeat!

Django - Reverse for '' not found. '' is not a valid view function or pattern name

I was receiving the same error when not specifying the app name before pattern name. In my case:

app-name : Blog

pattern-name : post-delete

reverse_lazy('Blog:post-delete') worked.

Where can I find the TypeScript version installed in Visual Studio?

You can do npm list | grep typescript if it's installed through npm.

Comparing strings in Java

did the same here needed to show "success" twice response is data from PHP

 String res=response.toString().trim;

                            Toast.makeText(sign_in.this,res,Toast.LENGTH_SHORT).show();
    if ( res.compareTo("success")==0){
    Toast.makeText(this,res,Toast.LENGTH_SHORT).show();
    }

Allow user to select camera or gallery for image

this code will help you, in that there is two button one for Camera and another for Gallery, and Image will be displayed in ImageView

https://github.com/siddhpuraamitr/Choose-Image-From-Gallery-Or-Camera

Is background-color:none valid CSS?

CSS Level 3 specifies the unset property value. From MDN:

The unset CSS keyword is the combination of the initial and inherit keywords. Like these two other CSS-wide keywords, it can be applied to any CSS property, including the CSS shorthand all. This keyword resets the property to its inherited value if it inherits from its parent or to its initial value if not. In other words, it behaves like the inherit keyword in the first case and like the initial keyword in the second case.

Unfortunately this value is currently not supported in all browsers, including IE, Safari and Opera. I suggest using transparent for the time being.

How to solve ADB device unauthorized in Android ADB host device?

Try this steps:

  1. unplug device
  2. adb kill-server
  3. adb start-server
  4. plug device

You need to allow Allow USB debugging in your device when popup.

Why are iframes considered dangerous and a security risk?

I'm assuming cross-domain iFrame since presumably the risk would be lower if you controlled it yourself.

  • Clickjacking is a problem if your site is included as an iframe
  • A compromised iFrame could display malicious content (imagine the iFrame displaying a login box instead of an ad)
  • An included iframe can make certain JS calls like alert and prompt which could annoy your user
  • An included iframe can redirect via location.href (yikes, imagine a 3p frame redirecting the customer from bankofamerica.com to bankofamerica.fake.com)
  • Malware inside the 3p frame (java/flash/activeX) could infect your user

C# removing items from listbox

protected void lbAddtoDestination_Click(object sender, EventArgs e)
        {
            AddRemoveItemsListBox(lstSourceSkills, lstDestinationSkills);
        }
        protected void lbRemovefromDestination_Click(object sender, EventArgs e)
        {
            AddRemoveItemsListBox(lstDestinationSkills, lstSourceSkills);
        }
        private void AddRemoveItemsListBox(ListBox source, ListBox destination)
        {
            List<ListItem> toBeRemoved = new List<ListItem>();
            foreach (ListItem item in source.Items)
            {
                if (item.Selected)
                {
                    toBeRemoved.Add(item);
                    destination.Items.Add(item);
                }
            }
            foreach (ListItem item in toBeRemoved) source.Items.Remove(item);
        }

Check if Nullable Guid is empty in c#

Beginning with C# 7.1, you can use default literal to produce the default value of a type when the compiler can infer the expression type.

Console.Writeline(default(Guid));  
   // ouptut: 00000000-0000-0000-0000-000000000000

Console.WriteLine(default(int));  // output: 0

Console.WriteLine(default(object) is null);  // output: True

https://docs.microsoft.com/en-us/dotnet/csharp/language-reference/operators/default

How to define a Sql Server connection string to use in VB.NET?

Try

Dim connectionString AS String = "Server=my_server;Database=name_of_db;User Id=user_name;Password=my_password"

And replace my_server, name_of_db, user_name and my_password with your values.

then Using sqlCon = New SqlConnection(connectionString) should work

also I think your SQL is wrong, it should be SET clickCount= clickCount + 1 I think.

And on a general note, the page you link to has a link called Connection String which shows you how to do this.

How to reset / remove chrome's input highlighting / focus border?

Problem is when you already have an outline. Chrome still changes something and it's a real pain. I cannot find what to change :

.search input {
  outline: .5em solid black;
  width:41%;
  background-color:white;
  border:none;
  font-size:1.4em;
  padding: 0 0.5em 0 0.5em;
  border-radius:3px;
  margin:0;
  height:2em;
}

.search input:focus, .search input:hover {
  outline: .5em solid black !important;
  box-shadow:none;
  border-color:transparent;;
 }

.search button {
  border:none;
  outline: .5em solid black;
  color:white;
  font-size:1.4em;
  padding: 0 0.9em 0 0.9em;
  border-radius: 3px;
  margin:0;
  height:2em;
  background: -webkit-gradient(linear, left top, left bottom, from(#4EB4F8), to(#4198DE));
  background: -webkit-linear-gradient(#4EB4F8, #4198DE);
  background: -moz-linear-gradient(top, #4EB4F8, #4198DE);
  background: -ms-linear-gradient(#4EB4F8, #4198DE);
  background: -o-linear-gradient(#4EB4F8, #4198DE);
  background: linear-gradient(#4EB4F8, #4198DE);
  filter: progid:DXImageTransform.Microsoft.gradient(startColorstr='#4EB4F8', endColorstr='#4198DE');
  zoom: 1;
}

No focus With focus

Remove a specific string from an array of string

Define "remove".

Arrays are fixed length and can not be resized once created. You can set an element to null to remove an object reference;

for (int i = 0; i < myStringArray.length(); i++)
{
    if (myStringArray[i].equals(stringToRemove))
    {
        myStringArray[i] = null;
        break;
    }
}

or

myStringArray[indexOfStringToRemove] = null;

If you want a dynamically sized array where the object is actually removed and the list (array) size is adjusted accordingly, use an ArrayList<String>

myArrayList.remove(stringToRemove); 

or

myArrayList.remove(indexOfStringToRemove);

Edit in response to OP's edit to his question and comment below

String r = myArrayList.get(rgenerator.nextInt(myArrayList.size()));

How do I use DateTime.TryParse with a Nullable<DateTime>?

Alternatively, if you are not concerned with the possible exception raised, you could change TryParse for Parse:

DateTime? d = DateTime.Parse("some valid text");

Although there won't be a boolean indicating success either, it could be practical in some situations where you know that the input text will always be valid.

Java: Find .txt files in specified folder

import org.apache.commons.io.FileUtils;   

List<File> htmFileList = new ArrayList<File>();

for (File file : (List<File>) FileUtils.listFiles(new File(srcDir), new String[]{"txt", "TXT"}, true)) {
    htmFileList.add(file);
}

This is my latest code to add all text files from a directory

JavaScript post request like a form submit

If you have Prototype installed, you can tighten up the code to generate and submit the hidden form like this:

 var form = new Element('form',
                        {method: 'post', action: 'http://example.com/'});
 form.insert(new Element('input',
                         {name: 'q', value: 'a', type: 'hidden'}));
 $(document.body).insert(form);
 form.submit();

How do I compare version numbers in Python?

Use packaging.version.parse.

>>> from packaging import version
>>> version.parse("2.3.1") < version.parse("10.1.2")
True
>>> version.parse("1.3.a4") < version.parse("10.1.2")
True
>>> isinstance(version.parse("1.3.a4"), version.Version)
True
>>> isinstance(version.parse("1.3.xy123"), version.LegacyVersion)
True
>>> version.Version("1.3.xy123")
Traceback (most recent call last):
...
packaging.version.InvalidVersion: Invalid version: '1.3.xy123'

packaging.version.parse is a third-party utility but is used by setuptools (so you probably already have it installed) and is conformant to the current PEP 440; it will return a packaging.version.Version if the version is compliant and a packaging.version.LegacyVersion if not. The latter will always sort before valid versions.

Note: packaging has recently been vendored into setuptools.


An ancient alternative still used by a lot of software is distutils.version, built in but undocumented and conformant only to the superseded PEP 386;

>>> from distutils.version import LooseVersion, StrictVersion
>>> LooseVersion("2.3.1") < LooseVersion("10.1.2")
True
>>> StrictVersion("2.3.1") < StrictVersion("10.1.2")
True
>>> StrictVersion("1.3.a4")
Traceback (most recent call last):
...
ValueError: invalid version number '1.3.a4'

As you can see it sees valid PEP 440 versions as “not strict” and therefore doesn’t match modern Python’s notion of what a valid version is.

As distutils.version is undocumented, here's the relevant docstrings.

Best way to structure a tkinter application?

This isn't a bad structure; it will work just fine. However, you do have to have functions in a function to do commands when someone clicks on a button or something

So what you could do is write classes for these then have methods in the class that handle commands for the button clicks and such.

Here's an example:

import tkinter as tk

class Window1:
    def __init__(self, master):
        pass
        # Create labels, entries,buttons
    def button_click(self):
        pass
        # If button is clicked, run this method and open window 2


class Window2:
    def __init__(self, master):
        #create buttons,entries,etc

    def button_method(self):
        #run this when button click to close window
        self.master.destroy()

def main(): #run mianloop 
    root = tk.Tk()
    app = Window1(root)
    root.mainloop()

if __name__ == '__main__':
    main()

Usually tk programs with multiple windows are multiple big classes and in the __init__ all the entries, labels etc are created and then each method is to handle button click events

There isn't really a right way to do it, whatever works for you and gets the job done as long as its readable and you can easily explain it because if you cant easily explain your program, there probably is a better way to do it.

Take a look at Thinking in Tkinter.

jQuery get specific option tag text

If you'd like to get the option with a value of 2, use

$("#list option[value='2']").text();

If you'd like to get whichever option is currently selected, use

$("#list option:selected").text();

Detect HTTP or HTTPS then force HTTPS in JavaScript

I like the answers for this question. But to be creative, I would like to share one more way:

<script>if (document.URL.substring(0,5) == "http:") window.location.replace('https:' + document.URL.substring(5));</script>

Add a common Legend for combined ggplots

@Giuseppe, you may want to consider this for a flexible specification of the plots arrangement (modified from here):

library(ggplot2)
library(gridExtra)
library(grid)

grid_arrange_shared_legend <- function(..., nrow = 1, ncol = length(list(...)), position = c("bottom", "right")) {

  plots <- list(...)
  position <- match.arg(position)
  g <- ggplotGrob(plots[[1]] + theme(legend.position = position))$grobs
  legend <- g[[which(sapply(g, function(x) x$name) == "guide-box")]]
  lheight <- sum(legend$height)
  lwidth <- sum(legend$width)
  gl <- lapply(plots, function(x) x + theme(legend.position = "none"))
  gl <- c(gl, nrow = nrow, ncol = ncol)

  combined <- switch(position,
                     "bottom" = arrangeGrob(do.call(arrangeGrob, gl),
                                            legend,
                                            ncol = 1,
                                            heights = unit.c(unit(1, "npc") - lheight, lheight)),
                     "right" = arrangeGrob(do.call(arrangeGrob, gl),
                                           legend,
                                           ncol = 2,
                                           widths = unit.c(unit(1, "npc") - lwidth, lwidth)))
  grid.newpage()
  grid.draw(combined)

}

Extra arguments nrow and ncol control the layout of the arranged plots:

dsamp <- diamonds[sample(nrow(diamonds), 1000), ]
p1 <- qplot(carat, price, data = dsamp, colour = clarity)
p2 <- qplot(cut, price, data = dsamp, colour = clarity)
p3 <- qplot(color, price, data = dsamp, colour = clarity)
p4 <- qplot(depth, price, data = dsamp, colour = clarity)
grid_arrange_shared_legend(p1, p2, p3, p4, nrow = 1, ncol = 4)
grid_arrange_shared_legend(p1, p2, p3, p4, nrow = 2, ncol = 2)

enter image description here enter image description here

Maximum execution time in phpMyadmin

Your change should work. However, there are potentially few php.ini configuration files with the 'xampp' stack. Try to identify whether or not there's an 'apache' specific php.ini. One potential location is:

C:\xampp\apache\bin\php.ini

Java 8 - Difference between Optional.flatMap and Optional.map

What helped me was a look at the source code of the two functions.

Map - wraps the result in an Optional.

public<U> Optional<U> map(Function<? super T, ? extends U> mapper) {
    Objects.requireNonNull(mapper);
    if (!isPresent())
        return empty();
    else {
        return Optional.ofNullable(mapper.apply(value)); //<--- wraps in an optional
    }
}

flatMap - returns the 'raw' object

public<U> Optional<U> flatMap(Function<? super T, Optional<U>> mapper) {
    Objects.requireNonNull(mapper);
    if (!isPresent())
        return empty();
    else {
        return Objects.requireNonNull(mapper.apply(value)); //<---  returns 'raw' object
    }
}

How to find sitemap.xml path on websites?

Use Google Search Operators to find it for you

search google with the below code..

inurl:domain.com filetype:xml click on this to view sitemap search example

change domain.com to the domain you want to find the sitemap. this should list all the xml files listed for the given domain.. including all sitemaps :)

How to send a simple string between two programs using pipes?

A regular pipe can only connect two related processes. It is created by a process and will vanish when the last process closes it.

A named pipe, also called a FIFO for its behavior, can be used to connect two unrelated processes and exists independently of the processes; meaning it can exist even if no one is using it. A FIFO is created using the mkfifo() library function.

Example

writer.c

#include <fcntl.h>
#include <sys/stat.h>
#include <sys/types.h>
#include <unistd.h>

int main()
{
    int fd;
    char * myfifo = "/tmp/myfifo";

    /* create the FIFO (named pipe) */
    mkfifo(myfifo, 0666);

    /* write "Hi" to the FIFO */
    fd = open(myfifo, O_WRONLY);
    write(fd, "Hi", sizeof("Hi"));
    close(fd);

    /* remove the FIFO */
    unlink(myfifo);

    return 0;
}

reader.c

#include <fcntl.h>
#include <stdio.h>
#include <sys/stat.h>
#include <unistd.h>

#define MAX_BUF 1024

int main()
{
    int fd;
    char * myfifo = "/tmp/myfifo";
    char buf[MAX_BUF];

    /* open, read, and display the message from the FIFO */
    fd = open(myfifo, O_RDONLY);
    read(fd, buf, MAX_BUF);
    printf("Received: %s\n", buf);
    close(fd);

    return 0;
}

Note: Error checking was omitted from the above code for simplicity.

Programmatically stop execution of python script?

The exit() and quit() built in functions do just what you want. No import of sys needed.

Alternatively, you can raise SystemExit, but you need to be careful not to catch it anywhere (which shouldn't happen as long as you specify the type of exception in all your try.. blocks).

Javascript - Track mouse position

Just a simplified version of @T.J. Crowder and @RegarBoy's answers.

Less is more in my opinion.

Check out onmousemove event for more info about the event.

Image mouse tracker

There's a new value of posX and posY every time the mouse moves according to the horizontal and vertical coordinates.

<!DOCTYPE html>
    <html>
    <head>
      <meta charset="utf-8">
      <title>Example Mouse Tracker</title>
      <style>    
        body {height: 3000px;}
        .dot {width: 2px;height: 2px;background-color: black;position: absolute;}
      </style>
    </head>
    <body>
    <p>Mouse tracker</p>
    <script>
    onmousemove = function(e){
        //Logging purposes
        console.log("mouse location:", e.clientX, e.clientY);

        //meat and potatoes of the snippet
        var pos = e;
        var dot;
        dot = document.createElement('div');
        dot.className = "dot";
        dot.style.left = pos.x + "px";
        dot.style.top = pos.y + "px";
        document.body.appendChild(dot);
    }      
    </script>
    </body>
    </html>

Convert Mercurial project to Git

You can try using fast-export:

cd ~
git clone https://github.com/frej/fast-export.git
git init git_repo
cd git_repo
~/fast-export/hg-fast-export.sh -r /path/to/old/mercurial_repo
git checkout HEAD

Also have a look at this SO question.


If you're using Mercurial version below 4.6, adrihanu got your back:

As he stated in his comment: "In case you use Mercurial < 4.6 and you got "revsymbol not found" error. You need to update your Mercurial or downgrade fast-export by running git checkout tags/v180317 inside ~/fast-export directory.".

Forcing a postback

You can use a data-bound control like the Repeater or ListView, re-bind it to a list of control properties as needed, and let it generate the controls dynamically.

As an alternative, you can use Response.Redirect(".") to re-load the same page.

How to specify the current directory as path in VBA?

I thought I had misunderstood but I was right. In this scenario, it will be ActiveWorkbook.Path

But the main issue was not here. The problem was with these 2 lines of code

strFile = Dir(strPath & "*.csv")

Which should have written as

strFile = Dir(strPath & "\*.csv")

and

With .QueryTables.Add(Connection:="TEXT;" & strPath & strFile, _

Which should have written as

With .QueryTables.Add(Connection:="TEXT;" & strPath & "\" & strFile, _

Using Environment Variables with Vue.js

For those using Vue CLI 3 and the webpack-simple install, Aaron's answer did work for me however I wasn't keen on adding my environment variables to my webpack.config.js as I wanted to commit it to GitHub. Instead I installed the dotenv-webpack plugin and this appears to load environment variables fine from a .env file at the root of the project without the need to prepend VUE_APP_ to the environment variables.

Comparing two integer arrays in Java

Here my approach,it may be useful to others.

public static void compareArrays(int[] array1, int[] array2) {
    if (array1.length != array2.length)
    {
           System.out.println("Not Equal");
    }
    else
    {
        int temp = 0;
        for (int i = 0; i < array2.length; i++) {  //Take any one of the array size
            temp^ = array1[i] ^ array2[i];   //with help of xor operator to find two array are equal or not                 
        }
        if( temp == 0 )
        {
             System.out.println("Equal");
        }
        else{
             System.out.println("Not Equal");
        }
    }
}

Function or sub to add new row and data to table

I needed this same solution, but if you use the native ListObject.Add() method then you avoid the risk of clashing with any data immediately below the table. The below routine checks the last row of the table, and adds the data in there if it's blank; otherwise it adds a new row to the end of the table:

Sub AddDataRow(tableName As String, values() As Variant)
    Dim sheet As Worksheet
    Dim table As ListObject
    Dim col As Integer
    Dim lastRow As Range

    Set sheet = ActiveWorkbook.Worksheets("Sheet1")
    Set table = sheet.ListObjects.Item(tableName)

    'First check if the last row is empty; if not, add a row
    If table.ListRows.Count > 0 Then
        Set lastRow = table.ListRows(table.ListRows.Count).Range
        For col = 1 To lastRow.Columns.Count
            If Trim(CStr(lastRow.Cells(1, col).Value)) <> "" Then
                table.ListRows.Add
                Exit For
            End If
        Next col
    Else
        table.ListRows.Add
    End If

    'Iterate through the last row and populate it with the entries from values()
    Set lastRow = table.ListRows(table.ListRows.Count).Range
    For col = 1 To lastRow.Columns.Count
        If col <= UBound(values) + 1 Then lastRow.Cells(1, col) = values(col - 1)
    Next col
End Sub

To call the function, pass the name of the table and an array of values, one value per column. You can get / set the name of the table from the Design tab of the ribbon, in Excel 2013 at least: enter image description here

Example code for a table with three columns:

Dim x(2)
x(0) = 1
x(1) = "apple"
x(2) = 2
AddDataRow "Table1", x

What does it mean to have an index to scalar variable error? python

exponent is a 1D array. This means that exponent[0] is a scalar, and exponent[0][i] is trying to access it as if it were an array.

Did you mean to say:

L = identity(len(l))
for i in xrange(len(l)):
    L[i][i] = exponent[i]

or even

L = diag(exponent)

?

jQuery get the image src

In my case this format worked on latest version of jQuery:

$('img#post_image_preview').src;

Access a function variable outside the function without using "global"

You could do something along this lines:

def static_example():
   if not hasattr(static_example, "static_var"):
       static_example.static_var = 0
   static_example.static_var += 1
   return static_example.static_var

print static_example()
print static_example()
print static_example()

.NET: Simplest way to send POST with data and read response

Given other answers are a few years old, currently here are my thoughts that may be helpful:

Simplest way

private async Task<string> PostAsync(Uri uri, HttpContent dataOut)
{
    var client = new HttpClient();
    var response = await client.PostAsync(uri, dataOut);
    return await response.Content.ReadAsStringAsync();
    // For non strings you can use other Content.ReadAs...() method variations
}

A More Practical Example

Often we are dealing with known types and JSON, so you can further extend this idea with any number of implementations, such as:

public async Task<T> PostJsonAsync<T>(Uri uri, object dtoOut)
{
    var content = new StringContent(JsonConvert.SerializeObject(dtoOut));
    content.Headers.ContentType = MediaTypeHeaderValue.Parse("application/json");

    var results = await PostAsync(uri, content); // from previous block of code

    return JsonConvert.DeserializeObject<T>(results); // using Newtonsoft.Json
}

An example of how this could be called:

var dataToSendOutToApi = new MyDtoOut();
var uri = new Uri("https://example.com");
var dataFromApi = await PostJsonAsync<MyDtoIn>(uri, dataToSendOutToApi);

Label axes on Seaborn Barplot

Seaborn's barplot returns an axis-object (not a figure). This means you can do the following:

import pandas as pd
import seaborn as sns
import matplotlib.pyplot as plt

fake = pd.DataFrame({'cat': ['red', 'green', 'blue'], 'val': [1, 2, 3]})
ax = sns.barplot(x = 'val', y = 'cat', 
              data = fake, 
              color = 'black')
ax.set(xlabel='common xlabel', ylabel='common ylabel')
plt.show()

Iterate over object keys in node.js

adjust his code:

Object.prototype.each = function(iterateFunc) {
        var counter = 0,
keys = Object.keys(this),
currentKey,
len = keys.length;
        var that = this;
        var next = function() {

            if (counter < len) {
                currentKey = keys[counter++];
                iterateFunc(currentKey, that[currentKey]);

                next();
            } else {
                that = counter = keys = currentKey = len = next = undefined;
            }
        };
        next();
    };

    ({ property1: 'sdsfs', property2: 'chat' }).each(function(key, val) {
        // do things
        console.log(key);
    });

How do I use 3DES encryption/decryption in Java?

Here's a very simply static encrypt/decrypt class biased on the Bouncy Castle no padding example by Jose Luis Montes de Oca. This one is using "DESede/ECB/PKCS7Padding" so I don't have to bother manually padding.


    package com.zenimax.encryption;

    import java.security.InvalidKeyException;
    import java.security.NoSuchAlgorithmException;
    import java.security.NoSuchProviderException;
    import java.security.Security;
    import javax.crypto.BadPaddingException;
    import javax.crypto.Cipher;
    import javax.crypto.IllegalBlockSizeException;
    import javax.crypto.NoSuchPaddingException;
    import javax.crypto.SecretKey;
    import javax.crypto.spec.SecretKeySpec;
    import org.bouncycastle.jce.provider.BouncyCastleProvider;

    /**
     * 
     * @author Matthew H. Wagner
     */
    public class TripleDesBouncyCastle {
        private static String TRIPLE_DES_TRANSFORMATION = "DESede/ECB/PKCS7Padding";
        private static String ALGORITHM = "DESede";
        private static String BOUNCY_CASTLE_PROVIDER = "BC";

        private static void init()
        {
            Security.addProvider(new BouncyCastleProvider());
        }

        public static byte[] encode(byte[] input, byte[] key)
                throws IllegalBlockSizeException, BadPaddingException,
                NoSuchAlgorithmException, NoSuchProviderException,
                NoSuchPaddingException, InvalidKeyException {
            init();
            SecretKey keySpec = new SecretKeySpec(key, ALGORITHM);
            Cipher encrypter = Cipher.getInstance(TRIPLE_DES_TRANSFORMATION,
                    BOUNCY_CASTLE_PROVIDER);
            encrypter.init(Cipher.ENCRYPT_MODE, keySpec);
            return encrypter.doFinal(input);
        }

        public static byte[] decode(byte[] input, byte[] key)
                throws IllegalBlockSizeException, BadPaddingException,
                NoSuchAlgorithmException, NoSuchProviderException,
                NoSuchPaddingException, InvalidKeyException {
            init();
            SecretKey keySpec = new SecretKeySpec(key, ALGORITHM);
            Cipher decrypter = Cipher.getInstance(TRIPLE_DES_TRANSFORMATION,
                    BOUNCY_CASTLE_PROVIDER);
            decrypter.init(Cipher.DECRYPT_MODE, keySpec);
            return decrypter.doFinal(input);
        }
    }

How to get the return value from a thread in python?

join always return None, i think you should subclass Thread to handle return codes and so.

How to add Active Directory user group as login in SQL Server

Here is my observation. I created a login (readonly) for a group windows(AD) user account but, its acting defiantly in different SQL servers. In the SQl servers that users can not see the databases I added view definition checked and also gave database execute permeation to the master database for avoiding error 229. I do not have this issue if I create a login for a user.

"Connection for controluser as defined in your configuration failed" with phpMyAdmin in XAMPP

on ubuntu /etc/phpmyadmin/config-db.php

make sure the password matches your config.inc.php for the control user

also for the blowfish too short error

edit /var/lib/phpmyadmin/blowfish_secret.inc.php and make the key longer

How can I remove or replace SVG content?

You should use append("svg:svg"), not append("svg") so that D3 makes the element with the correct 'namespace' if you're using xhtml.

Binding ItemsSource of a ComboBoxColumn in WPF DataGrid

Pls, check if DataGridComboBoxColumn xaml below would work for you:

<DataGridComboBoxColumn 
    SelectedValueBinding="{Binding CompanyID}" 
    DisplayMemberPath="Name" 
    SelectedValuePath="ID">

    <DataGridComboBoxColumn.ElementStyle>
        <Style TargetType="{x:Type ComboBox}">
            <Setter Property="ItemsSource" Value="{Binding Path=DataContext.CompanyItems, RelativeSource={RelativeSource AncestorType={x:Type Window}}}" />
        </Style>
    </DataGridComboBoxColumn.ElementStyle>
    <DataGridComboBoxColumn.EditingElementStyle>
        <Style TargetType="{x:Type ComboBox}">
            <Setter Property="ItemsSource" Value="{Binding Path=DataContext.CompanyItems, RelativeSource={RelativeSource AncestorType={x:Type Window}}}" />
        </Style>
    </DataGridComboBoxColumn.EditingElementStyle>
</DataGridComboBoxColumn>

Here you can find another solution for the problem you're facing: Using combo boxes with the WPF DataGrid

Mongoose: findOneAndUpdate doesn't return updated document

If you want to return the altered document you need to set the option {new:true} API reference you can use Cat.findOneAndUpdate(conditions, update, options, callback) // executes

Taken by the official Mongoose API http://mongoosejs.com/docs/api.html#findoneandupdate_findOneAndUpdate you can use the following parameters

A.findOneAndUpdate(conditions, update, options, callback) // executes
A.findOneAndUpdate(conditions, update, options)  // returns Query
A.findOneAndUpdate(conditions, update, callback) // executes
A.findOneAndUpdate(conditions, update)           // returns Query
A.findOneAndUpdate()                             // returns Query

Another implementation thats is not expressed in the official API page and is what I prefer to use is the Promise base implementation that allow you to have .catch where you can deal with all your various error there.

    let cat: catInterface = {
        name: "Naomi"
    };

    Cat.findOneAndUpdate({age:17}, cat,{new: true}).then((data) =>{
        if(data === null){
            throw new Error('Cat Not Found');
        }
        res.json({ message: 'Cat updated!' })
        console.log("New cat data", data);
    }).catch( (error) => {
        /*
            Deal with all your errors here with your preferred error handle middleware / method
         */
        res.status(500).json({ message: 'Some Error!' })
        console.log(error);
    });

Setting up Vim for Python

There is a bundled collection of Vim plugins for Python development: http://www.vim.org/scripts/script.php?script_id=3770

Python: print a generator expression?

>>> list(x for x in string.letters if x in (y for y in "BigMan on campus"))
['a', 'c', 'g', 'i', 'm', 'n', 'o', 'p', 's', 'u', 'B', 'M']

Run Excel Macro from Outside Excel Using VBScript From Command Line

I tried to adapt @Siddhart's code to a relative path to run my open_form macro, but it didn't seem to work. Here was my first attempt. My working solution is below.

Option Explicit

Dim xlApp, xlBook
dim fso
dim curDir
set fso = CreateObject("Scripting.FileSystemObject")
curDir = fso.GetAbsolutePathName(".")
set fso = nothing

Set xlApp = CreateObject("Excel.Application")
'~~> Change Path here
Set xlBook = xlApp.Workbooks.Open(curDir & "Excels\CLIENTES.xlsb", 0, true)
xlApp.Run "open_form"
xlBook.Close
xlApp.Quit

Set xlBook = Nothing
Set xlApp = Nothing

WScript.Echo "Finished."

EDIT

I have actually worked it out, just in case someone wants to run a userform "alike" a stand alone application:

Issues I was facing:

1 - I did not want to use the Workbook_Open Event as the excel is locked in read only. 2 - The batch command is limited that the fact that (to my knowledge) it cannot call the macro.

I first wrote a macro to launch my userform while hiding the application:

Sub open_form()
 Application.Visible = False
 frmAddClient.Show vbModeless
End Sub

I then created a vbs to launch this macro (doing it with a relative path has been tricky):

dim fso
dim curDir
dim WinScriptHost
set fso = CreateObject("Scripting.FileSystemObject")
curDir = fso.GetAbsolutePathName(".")
set fso = nothing

Set xlObj = CreateObject("Excel.application")
xlObj.Workbooks.Open curDir & "\Excels\CLIENTES.xlsb"
xlObj.Run "open_form"

And I finally did a batch file to execute the VBS...

@echo off
pushd %~dp0
cscript Add_Client.vbs

Note that I have also included the "Set back to visible" in my Userform_QueryClose:

Private Sub cmdClose_Click()
Unload Me
End Sub

Private Sub UserForm_QueryClose(Cancel As Integer, CloseMode As Integer)
    ThisWorkbook.Close SaveChanges:=True
    Application.Visible = True
    Application.Quit
End Sub

Anyway, thanks for your help, and I hope this will help if someone needs it

What is a simple command line program or script to backup SQL server databases?

I found this on a Microsoft Support page http://support.microsoft.com/kb/2019698.

It works great! And since it came from Microsoft, I feel like it's pretty legit.

Basically there are two steps.

  1. Create a stored procedure in your master db. See msft link or if it's broken try here: http://pastebin.com/svRLkqnq
  2. Schedule the backup from your task scheduler. You might want to put into a .bat or .cmd file first and then schedule that file.

    sqlcmd -S YOUR_SERVER_NAME\SQLEXPRESS -E -Q "EXEC sp_BackupDatabases @backupLocation='C:\SQL_Backup\', @backupType='F'"  1>c:\SQL_Backup\backup.log            
    

Obviously replace YOUR_SERVER_NAME with your computer name or optionally try .\SQLEXPRESS and make sure the backup folder exists. In this case it's trying to put it into c:\SQL_Backup

MySQL: Insert datetime into other datetime field

Try

    UPDATE products SET former_date=20111218131717 WHERE id=1

Alternatively, you might want to look at using the STR_TO_DATE (see STR_TO_DATE(str,format)) function.

sqlite3.ProgrammingError: Incorrect number of bindings supplied. The current statement uses 1, and there are 74 supplied

You need to pass in a sequence, but you forgot the comma to make your parameters a tuple:

cursor.execute('INSERT INTO images VALUES(?)', (img,))

Without the comma, (img) is just a grouped expression, not a tuple, and thus the img string is treated as the input sequence. If that string is 74 characters long, then Python sees that as 74 separate bind values, each one character long.

>>> len(img)
74
>>> len((img,))
1

If you find it easier to read, you can also use a list literal:

cursor.execute('INSERT INTO images VALUES(?)', [img])

Access denied for root user in MySQL command-line

Server file only change name folder

etc/mysql
rename
mysql-

Detach (move) subdirectory into separate Git repository

Paul's answer creates a new repository containing /ABC, but does not remove /ABC from within /XYZ. The following command will remove /ABC from within /XYZ:

git filter-branch --tree-filter "rm -rf ABC" --prune-empty HEAD

Of course, test it in a 'clone --no-hardlinks' repository first, and follow it with the reset, gc and prune commands Paul lists.

Cannot implicitly convert type 'System.DateTime?' to 'System.DateTime'. An explicit conversion exists

you should be using the .Value of the datetime parameter. All Nullable structs have a value property which returns the concrete type of the object. but you must check to see if it is null beforehand otherwise you will get a runtime error.

i.e:

datetime.Value

but check to see if it has a value first!

if (datetime.HasValue)
{
   // work with datetime.Value
}

How to test web service using command line curl

In addition to existing answers it is often desired to format the REST output (typically JSON and XML lacks indentation). Try this:

$ curl https://api.twitter.com/1/help/configuration.xml  | xmllint --format -
$ curl https://api.twitter.com/1/help/configuration.json | python -mjson.tool

Tested on Ubuntu 11.0.4/11.10.

Another issue is the desired content type. Twitter uses .xml/.json extension, but more idiomatic REST would require Accept header:

$ curl -H "Accept: application/json"

Creating a LINQ select from multiple tables

change

select op) 

to

select new { op, pg })

How to get the parent dir location

Use the following to jump to previous folder:

os.chdir(os.pardir)

If you need multiple jumps a good and easy solution will be to use a simple decorator in this case.

Official reasons for "Software caused connection abort: socket write error"

The java.net.SocketException is thrown when there is an error creating or accessing a socket (such as TCP). This usually can be caused when the server has terminated the connection (without properly closing it), so before getting the full response. In most cases this can be caused either by the timeout issue (e.g. the response takes too much time or server is overloaded with the requests), or the client sent the SYN, but it didn't receive ACK (acknowledgment of the connection termination). For timeout issues, you can consider increasing the timeout value.

The Socket Exception usually comes with the specified detail message about the issue.

Example of detailed messages:

  • Software caused connection abort: recv failed.

    The error indicates an attempt to send the message and the connection has been aborted by your server. If this happened while connecting to the database, this can be related to using not compatible Connector/J JDBC driver.

    Possible solution: Make sure you've proper libraries/drivers in your CLASSPATH.

  • Software caused connection abort: connect.

    This can happen when there is a problem to connect to the remote. For example due to virus-checker rejecting the remote mail requests.

    Possible solution: Check Virus scan service whether it's blocking the port for the outgoing requests for connections.

  • Software caused connection abort: socket write error.

    Possible solution: Make sure you're writing the correct length of bytes to the stream. So double check what you're sending. See this thread.

  • Connection reset by peer: socket write error / Connection aborted by peer: socket write error

    The application did not check whether keep-alive connection had been timed out on the server side.

    Possible solution: Ensure that the HttpClient is non-null before reading from the connection.E13222_01

  • Connection reset by peer.

    The connection has been terminated by the peer (server).

  • Connection reset.

    The connection has been either terminated by the client or closed by the server end of the connection due to request with the request.

    See: What's causing my java.net.SocketException: Connection reset?

How to change the default port of mysql from 3306 to 3360

In Windows 8.1 x64 bit os, Currently I am using MySQL version :

Server version: 5.7.11-log MySQL Community Server (GPL)

For changing your MySQL port number, Go to installation directory, my installation directory is :

C:\Program Files\MySQL\MySQL Server 5.7

open the my-default.ini Configuration Setting file in any text editor.

search the line in the configuration file.

# port = .....

replace it with :

port=<my_new_port_number>

like my self changed to :

port=15800

To apply the changes don't forget to immediate either restart the MySQL Server or your OS.

Hope this would help many one.

Apache Server (xampp) doesn't run on Windows 10 (Port 80)

I had the same problem on windows 10, IIS/10.0 was using port 80

To solve that:

  • find service "W3SVC"
  • disable it, or set it to "manual"

French name is: "Service de publication World Wide Web"

English name is: "World Wide Web Publishing Service"

german name is: "WWW-Publishingdienst" – thanks @fiffy

Polish name is: "Usluga publikowania w sieci WWW" - thanks @KrzysDan

Russian name is "?????? ???-??????????" – thanks @Kreozot

Italian name is "Servizio Pubblicazione sul Web" – thanks @Claudio-Venturini

Español name is "Servicio de publicación World Wide Web" - thanks @Daniel-Santarriaga

Portuguese (Brazil) name is "Serviço de publicação da World Wide Web" - thanks @thiago-born


Alternatives :


Edit 07 oct 2015: For more details, see Matthew Stumphy's answer Apache Server (xampp) doesn't run on Windows 10 (Port 80)

Force browser to clear cache

There is one trick that can be used.The trick is to append a parameter/string to the file name in the script tag and change it when you file changes.

<script src="myfile.js?version=1.0.0"></script>

The browser interprets the whole string as the file path even though what comes after the "?" are parameters. So wat happens now is that next time when you update your file just change the number in the script tag on your website (Example <script src="myfile.js?version=1.0.1"></script>) and each users browser will see the file has changed and grab a new copy.

how to call scalar function in sql server 2008

For Scalar Function Syntax is

Select dbo.Function_Name(parameter_name)

Select dbo.Department_Employee_Count('HR')

How to disable an input box using angular.js

<input data-ng-model="userInf.username"  class="span12 editEmail" type="text"  placeholder="[email protected]"  pattern="[^@]+@[^@]+\.[a-zA-Z]{2,6}" required ng-disabled="true"/>

Check status of one port on remote host

In Command Prompt, you can use the command telnet.. For Example, to connect to IP 192.168.10.1 with port 80,

telnet 192.168.10.1 80

To enable telnet in Windows 7 and above click. From the linked article, enable telnet through control panel -> programs and features -> windows features -> telnet client, or just run this in an admin prompt:

dism /online /Enable-Feature /FeatureName:TelnetClient

Print debugging info from stored procedure in MySQL

I usually create log table with a stored procedure to log to it. The call the logging procedure wherever needed from the procedure under development.

Looking at other posts on this same question, it seems like a common practice, although there are some alternatives.

Add "Are you sure?" to my excel button, how can I?

On your existing button code, simply insert this line before the procedure:

If MsgBox("This will erase everything! Are you sure?", vbYesNo) = vbNo Then Exit Sub

This will force it to quit if the user presses no.

How do I determine the size of my array in C?

int size = (&arr)[1] - arr;

Check out this link for explanation

How to pull specific directory with git

Sometimes, you just want to have a look at previous copies of files without the rigmarole of going through the diffs.

In such a case, it's just as easy to make a clone of a repository and checkout the specific commit that you are interested in and have a look at the subdirectory in that cloned repository. Because everything is local you can just delete this clone when you are done.

best OCR (Optical character recognition) example in android

Like you I also faced many problems implementing OCR in Android, but after much Googling I found the solution, and it surely is the best example of OCR.

Let me explain using step-by-step guidance.

First, download the source code from https://github.com/rmtheis/tess-two.

Import all three projects. After importing you will get an error. To solve the error you have to create a res folder in the tess-two project

enter image description here

First, just create res folder in tess-two by tess-two->RightClick->new Folder->Name it "res"

After doing this in all three project the error should be gone.

Now download the source code from https://github.com/rmtheis/android-ocr, here you will get best example.

Now you just need to import it into your workspace, but first you have to download android-ndk from this site:

http://developer.android.com/tools/sdk/ndk/index.html i have windows 7 - 32 bit PC so I have download http://dl.google.com/android/ndk/android-ndk-r9-windows-x86.zip this file

Now extract it suppose I have extract it into E:\Software\android-ndk-r9 so I will set this path on Environment Variable

Right Click on MyComputer->Property->Advance-System-Settings->Advance->Environment Variable-> find PATH on second below Box and set like path like below picture

enter image description here

done it

Now open cmd and go to on D:\Android Workspace\tess-two like below

enter image description here

If you have successfully set up environment variable of NDK then just type ndk-build just like above picture than enter you will not get any kind of error and all file will be compiled successfully:

Now download other source code also from https://github.com/rmtheis/tess-two , and extract and import it and give it name OCRTest, like in my PC which is in D:\Android Workspace\OCRTest

enter image description here

Import test-two in this and run OCRTest and run it; you will get the best example of OCR.

Get PostGIS version

As the above people stated, select PostGIS_full_version(); will answer your question. On my machine, where I'm running PostGIS 2.0 from trunk, I get the following output:

postgres=# select PostGIS_full_version();
postgis_full_version                                                                  
-------------------------------------------------------------------------------------------------------------------------------------------------------
POSTGIS="2.0.0alpha4SVN" GEOS="3.3.2-CAPI-1.7.2" PROJ="Rel. 4.7.1, 23 September 2009" GDAL="GDAL 1.8.1, released 2011/07/09" LIBXML="2.7.3" USE_STATS
(1 row)

You do need to care about the versions of PROJ and GEOS that are included if you didn't install an all-inclusive package - in particular, there's some brokenness in GEOS prior to 3.3.2 (as noted in the postgis 2.0 manual) in dealing with geometry validity.

Find integer index of rows with NaN in pandas dataframe

Don't know if this is too late but you can use np.where to find the indices of non values as such:

indices = list(np.where(df['b'].isna()[0]))

Enabling SSL with XAMPP

I finally got this to work on my own hosted xampp windows 10 server web site. I.e. padlocks came up as ssl. I am using xampp version from November 2020.

  1. Went to certbot.eff.org. Selected from their home page software [apache] and system [windows]. Then downloaded and installed certbot software found at the next page into my C drive.

  2. Then from command line [cmd in Windows Start and then before you open cmd right click to run cmd as admin] I enhtered the command from Certbot page above. I.e. navigated to system32-- C:\WINDOWS\system32> certbot certonly --standalone

  3. Then followed the prompts and enteredmy domain name. This created certs as cert1.pem and key1.pem in C:\Certbot yourwebsitedomain folder. the cmd windows tells you where these are.

  4. Then took these and changed their names from cert1.pem to my domainname or shorter+cert.pem and same for domainname or shorter+key.key. Copied these into C:\xampp\apache\ssl.crt and ssl.key folders respectively.

  5. Then for G:\xampp\apache\conf\extra\httpd-vhosts entered the following:

<VirtualHost *:443>
    DocumentRoot "G:/xampp/htdocs/yourwebsitedomainname.hopto.org/public/" ###NB My document root is public.  Yours may not be.  Or could have an index.php page before /public###
    ServerName yourwebsitedomainnamee.hopto.org 
    <Directory G:/xampp/htdocs/yourwebsitedomainname.hopto.org>
        Options Indexes FollowSymLinks Includes ExecCGI
        AllowOverride All
        Require all granted
    </Directory>
    ErrorLog "G:/xampp/apache/logs/error.log"
    CustomLog "G:/xampp/apache/logs/access.log" common
    SSLEngine on
SSLCertificateFile "G:\xampp\apache\conf\ssl.crt\abscert.pem"
SSLCertificateKeyFile "G:\xampp\apache\conf\ssl.key\abskey.pem"
</VirtualHost>  
     
  1. Then navigated to G:\xampp\apache\conf\extra\httpd-ssl.conf and did as was advised above. I missed this important step for days until I read this post. Thank you! I.e. entered
<VirtualHost _default_:443>
DocumentRoot "G:/xampp/htdocs/yourwebsitedomainnamee.hopto.org/public/"
###NB My document root is public.  Yours may not be.  Or could have an index.php page before /public###
SSLEngine on
SSLCertificateFile "conf/ssl.crt/abscert.pem"
SSLCertificateKeyFile "conf/ssl.key/abskey.pem"
CustomLog "G:/xampp/apache/logs/ssl_request.log" \
          "%t %h %{SSL_PROTOCOL}x %{SSL_CIPHER}x \"%r\" %b"
</VirtualHost>  

Note1. I used www.noip.com to register the domain name. Note2. Rather then try to get them to give me a ssl certificate, as I could not get it to work, the above worked instead. Note3 I use the noip DUC software to keep my personally hosted web site in sync with noip. Note4. Very important to stop and start xampp server after each change you make in xampp. If xampp fails for some reason instead of starting the xampp consol try the start xampp as this will give you problems you can bug fix. Copy these quickly and paste into note.txt.

How to call Stored Procedure in Entity Framework 6 (Code-First)?

Using your example, here are two ways to accomplish this:

1 - Use Stored procedure mapping

Note that this code will work with or without mapping. If you turn off mapping on the entity, EF will generate an insert + select statement.

protected void btnSave_Click(object sender, EventArgs e)
{
     using (var db = DepartmentContext() )
     {
        var department = new Department();

        department.Name = txtDepartment.text.trim();

        db.Departments.add(department);
        db.SaveChanges();

        // EF will populate department.DepartmentId
        int departmentID = department.DepartmentId;
     }
}

2 - Call the stored procedure directly

protected void btnSave_Click(object sender, EventArgs e)
{
     using (var db = DepartmentContext() )
     {
        var name = new SqlParameter("@name", txtDepartment.text.trim());

        //to get this to work, you will need to change your select inside dbo.insert_department to include name in the resultset
        var department = db.Database.SqlQuery<Department>("dbo.insert_department @name", name).SingleOrDefault();

       //alternately, you can invoke SqlQuery on the DbSet itself:
       //var department = db.Departments.SqlQuery("dbo.insert_department @name", name).SingleOrDefault();

        int departmentID = department.DepartmentId;
     }
}

I recommend using the first approach, as you can work with the department object directly and not have to create a bunch of SqlParameter objects.

Count number of times a date occurs and make a graph out of it

If you have Excel 2010 you can copy your data into another column, than select it and choose Data -> Remove Duplicates. You can then write =COUNTIF($A$1:$A$100,B1) next to it and copy the formula down. This assumes you have your values in range A1:A100 and the de-duplicated values are in column B.

jQuery .live() vs .on() method for adding a click event after loading dynamic html

.on() is for jQuery version 1.7 and above. If you have an older version, use this:

$("#SomeId").live("click",function(){
    //do stuff;
});

How to grep Git commit diffs or contents for a certain word?

git log's pickaxe will find commits with changes including "word" with git log -Sword

Are the shift operators (<<, >>) arithmetic or logical in C?

According to K&R 2nd edition the results are implementation-dependent for right shifts of signed values.

Wikipedia says that C/C++ 'usually' implements an arithmetic shift on signed values.

Basically you need to either test your compiler or not rely on it. My VS2008 help for the current MS C++ compiler says that their compiler does an arithmetic shift.

MySQL SELECT DISTINCT multiple columns

Taking a guess at the results you want so maybe this is the query you want then

SELECT DISTINCT a FROM my_table
UNION 
SELECT DISTINCT b FROM my_table
UNION
SELECT DISTINCT c FROM my_table
UNION
SELECT DISTINCT d FROM my_table

Create auto-numbering on images/figures in MS Word

  • Select whole document (Ctrl+A)
  • Press F9
  • Save

Should update the figure caption automatically.

My question is tho, how can one also 'assign' referenced figures '(Fig.4)' in the text to do the same thing - aka change when an image is added above it?

EDIT: Figured it out.. In word go to Insert and Cross-ref and assign the ref. Then Ctrl+A and F9 and everything should sort itself out.

How to switch between python 2.7 to python 3 from command line?

For Windows 7, I just rename the python.exe from the Python 3 folder to python3.exe and add the path into the environment variables. Using that, I can execute python test_script.py and the script runs with Python 2.7 and when I do python3 test_script.py, it runs the script in Python 3.

To add Python 3 to the environment variables, follow these steps -

  1. Right Click on My Computer and go to Properties.
  2. Go to Advanced System Settings.
  3. Click on Environment Variables and edit PATH and add the path to your Python 3 installation directory.

For example,

enter image description here

Is it possible to create a remote repo on GitHub from the CLI without opening browser?

For directions on creating a token, go here This is the command you will type (as of the date of this answer. (replace all CAPS keywords):

curl -u 'YOUR_USERNAME' -d '{"scopes":["repo"],"note":"YOUR_NOTE"}' https://api.github.com/authorizations

Once you enter your password you will see the following which contains your token.

{
  "app": {
    "name": "YOUR_NOTE (API)",
    "url": "http://developer.github.com/v3/oauth/#oauth-authorizations-api"
  },
  "note_url": null,
  "note": "YOUR_NOTE",
  "scopes": [
    "repo"
  ],
  "created_at": "2012-10-04T14:17:20Z",
  "token": "xxxxx",
  "updated_at": "2012-10-04T14:17:20Z",
  "id": xxxxx,
  "url": "https://api.github.com/authorizations/697577"
}

You can revoke your token anytime by going here

Running code in main thread from another thread

More precise Kotlin code using handler :

Handler(Looper.getMainLooper()).post {  
 // your codes here run on main Thread
 }

SSIS expression: convert date to string

If, like me, you are trying to use GETDATE() within an expression and have the seemingly unreasonable requirement (SSIS/SSDT seems very much a work in progress to me, and not a polished offering) of wanting that date to get inserted into SQL Server as a valid date (type = datetime), then I found this expression to work:

@[User::someVar] = (DT_WSTR,4)YEAR(GETDATE()) + "-"  + RIGHT("0" + (DT_WSTR,2)MONTH(GETDATE()), 2) + "-"  + RIGHT("0" + (DT_WSTR,2)DAY( GETDATE()), 2) + " " + RIGHT("0" + (DT_WSTR,2)DATEPART("hh", GETDATE()), 2) + ":" + RIGHT("0" + (DT_WSTR,2)DATEPART("mi", GETDATE()), 2) + ":" + RIGHT("0" + (DT_WSTR,2)DATEPART("ss", GETDATE()), 2)

I found this code snippet HERE

make: *** No rule to make target `all'. Stop

Your makefile should ideally be named makefile, not make. Note that you can call your makefile anything you like, but as you found, you then need the -f option with make to specify the name of the makefile. Using the default name of makefile just makes life easier.

Cannot change column used in a foreign key constraint

The type and definition of foreign key field and reference must be equal. This means your foreign key disallows changing the type of your field.

One solution would be this:

LOCK TABLES 
    favorite_food WRITE,
    person WRITE;

ALTER TABLE favorite_food
    DROP FOREIGN KEY fk_fav_food_person_id,
    MODIFY person_id SMALLINT UNSIGNED;

Now you can change you person_id

ALTER TABLE person MODIFY person_id SMALLINT UNSIGNED AUTO_INCREMENT;

recreate foreign key

ALTER TABLE favorite_food
    ADD CONSTRAINT fk_fav_food_person_id FOREIGN KEY (person_id)
          REFERENCES person (person_id);

UNLOCK TABLES;

EDIT: Added locks above, thanks to comments

You have to disallow writing to the database while you do this, otherwise you risk data integrity problems.

I've added a write lock above

All writing queries in any other session than your own ( INSERT, UPDATE, DELETE ) will wait till timeout or UNLOCK TABLES; is executed

http://dev.mysql.com/doc/refman/5.5/en/lock-tables.html

EDIT 2: OP asked for a more detailed explanation of the line "The type and definition of foreign key field and reference must be equal. This means your foreign key disallows changing the type of your field."

From MySQL 5.5 Reference Manual: FOREIGN KEY Constraints

Corresponding columns in the foreign key and the referenced key must have similar internal data types inside InnoDB so that they can be compared without a type conversion. The size and sign of integer types must be the same. The length of string types need not be the same. For nonbinary (character) string columns, the character set and collation must be the same.

How to get second-highest salary employees in a table

Below query can be used to find the nth maximum value, just replace 2 from nth number

select * from emp e1 where 2 =(select count(distinct(salary)) from emp e2
   where e2.emp >= e1.emp)

How to change Maven local repository in eclipse

In Eclipse Photon navigate to Windows > Preferences > Maven > User Settings > User Setting

For "User settings" Browse to the settings.xml of the maven. ex. in my case maven it is located on the path C:\Program Files\Apache Software Distribution\apache-maven-3.5.4\conf\Settings.xml

Depending on the Settings.xml the Local Repository gets automatically configured to the specified location. Click here for screenshot

Laravel 5 Failed opening required bootstrap/../vendor/autoload.php

I had same problem with laravel and artisan, the solution:

php artisan key:generate

Why doesn't importing java.util.* include Arrays and Lists?

I have just compile it and it compiles fine without the implicit import, probably you're seeing a stale cache or something of your IDE.

Have you tried compiling from the command line?

I have the exact same version:

here it is

Probably you're thinking the warning is an error.

UPDATE

It looks like you have a Arrays.class file in the directory where you're trying to compile ( probably created before ). That's why the explicit import solves the problem. Try copying your source code to a clean new directory and try again. You'll see there is no error this time. Or, clean up your working directory and remove the Arrays.class

Embed YouTube Video with No Ads

From the YouTube help:

You will automatically be opted into showing ads on embedded videos if you've associated your YouTube and AdSense accounts and have enabled your videos for embedding.

If you don't want to show overlay ads on your embedded videos, you can opt your videos out of showing overlay ads, though this will also disable overlay ads on your videos on YouTube.com. You may also disable your videos for embedding.

https://support.google.com/youtube/answer/132596?hl=en

Another technical solution could be to use a custom video player, and streamline the youtube video with that one. Have not tried but guess that the ads cannot be displayed in a custom player. However, could be forbidden.

Save attachments to a folder and rename them

This is my Save Attachments script. You select all the messages that you want the attachments saved from, and it will save a copy there. It also adds text to the message body indicating where the attachment is saved. You could easily change the folder name to include the date, but you would need to make sure the folder existed before starting to save files.

Public Sub SaveAttachments()
Dim objOL As Outlook.Application
Dim objMsg As Outlook.MailItem 'Object
Dim objAttachments As Outlook.Attachments
Dim objSelection As Outlook.Selection
Dim i As Long
Dim lngCount As Long
Dim strFile As String
Dim strFolderpath As String
Dim strDeletedFiles As String

' Get the path to your My Documents folder
strFolderpath = CreateObject("WScript.Shell").SpecialFolders(16)
On Error Resume Next

' Instantiate an Outlook Application object.
Set objOL = CreateObject("Outlook.Application")

' Get the collection of selected objects.
Set objSelection = objOL.ActiveExplorer.Selection

' Set the Attachment folder.
strFolderpath = strFolderpath & "\Attachments\"

' Check each selected item for attachments. If attachments exist,
' save them to the strFolderPath folder and strip them from the item.
For Each objMsg In objSelection

    ' This code only strips attachments from mail items.
    ' If objMsg.class=olMail Then
    ' Get the Attachments collection of the item.
    Set objAttachments = objMsg.Attachments
    lngCount = objAttachments.Count
    strDeletedFiles = ""

    If lngCount > 0 Then

        ' We need to use a count down loop for removing items
        ' from a collection. Otherwise, the loop counter gets
        ' confused and only every other item is removed.

        For i = lngCount To 1 Step -1

            ' Save attachment before deleting from item.
            ' Get the file name.
            strFile = objAttachments.Item(i).FileName

            ' Combine with the path to the Temp folder.
            strFile = strFolderpath & strFile

            ' Save the attachment as a file.
            objAttachments.Item(i).SaveAsFile strFile

            ' Delete the attachment.
            objAttachments.Item(i).Delete

            'write the save as path to a string to add to the message
            'check for html and use html tags in link
            If objMsg.BodyFormat <> olFormatHTML Then
                strDeletedFiles = strDeletedFiles & vbCrLf & "<file://" & strFile & ">"
            Else
                strDeletedFiles = strDeletedFiles & "<br>" & "<a href='file://" & _
                strFile & "'>" & strFile & "</a>"
            End If

            'Use the MsgBox command to troubleshoot. Remove it from the final code.
            'MsgBox strDeletedFiles

        Next i

        ' Adds the filename string to the message body and save it
        ' Check for HTML body
        If objMsg.BodyFormat <> olFormatHTML Then
            objMsg.Body = vbCrLf & "The file(s) were saved to " & strDeletedFiles & vbCrLf & objMsg.Body
        Else
            objMsg.HTMLBody = "<p>" & "The file(s) were saved to " & strDeletedFiles & "</p>" & objMsg.HTMLBody
        End If
        objMsg.Save
    End If
Next

ExitSub:

Set objAttachments = Nothing
Set objMsg = Nothing
Set objSelection = Nothing
Set objOL = Nothing
End Sub

How to split/partition a dataset into training and test datasets for, e.g., cross validation?

I wrote a function for my own project to do this (it doesn't use numpy, though):

def partition(seq, chunks):
    """Splits the sequence into equal sized chunks and them as a list"""
    result = []
    for i in range(chunks):
        chunk = []
        for element in seq[i:len(seq):chunks]:
            chunk.append(element)
        result.append(chunk)
    return result

If you want the chunks to be randomized, just shuffle the list before passing it in.

Count number of cells with any value (string or number) in a column in Google Docs Spreadsheet

Shorter and dealing with a column (entire, not just a section of a column):

=COUNTA(A:A)

COUNTA

Beware, a cell containing just a space would be included in the count.

python pandas remove duplicate columns

The way below will identify dupe columns to review what is going wrong building the dataframe originally.

dupes = pd.DataFrame(df.columns)
dupes[dupes.duplicated()]

What is web.xml file and what are all things can I do with it?

What all should I know about web.xml apart from element name and their usage ?

The SINGLE most important JSP configuration parameter of ALL TIME is in your web.xml. Ladies and gentlemen, I give you... the TRIM-DIRECTIVE-WHITESPACES option!

<jsp-config>
    <jsp-property-group>
        <url-pattern>*.jsp</url-pattern>
        <trim-directive-whitespaces>true</trim-directive-whitespaces>
    </jsp-property-group>
</jsp-config>

This removes all the hundreds or thousands of lines of white space that you'll get in your generated HTML if you use any tag libraries (loops are particularly ugly & wasteful).

The other big one is the default web page (the page you get automatically sent to when you don't enter a web page in the URL):

<welcome-file-list>
    <welcome-file>index.jsp</welcome-file>
</welcome-file-list>    

Timer function to provide time in nano seconds using C++

For C++11, here is a simple wrapper:

#include <iostream>
#include <chrono>

class Timer
{
public:
    Timer() : beg_(clock_::now()) {}
    void reset() { beg_ = clock_::now(); }
    double elapsed() const {
        return std::chrono::duration_cast<second_>
            (clock_::now() - beg_).count(); }

private:
    typedef std::chrono::high_resolution_clock clock_;
    typedef std::chrono::duration<double, std::ratio<1> > second_;
    std::chrono::time_point<clock_> beg_;
};

Or for C++03 on *nix,

class Timer
{
public:
    Timer() { clock_gettime(CLOCK_REALTIME, &beg_); }

    double elapsed() {
        clock_gettime(CLOCK_REALTIME, &end_);
        return end_.tv_sec - beg_.tv_sec +
            (end_.tv_nsec - beg_.tv_nsec) / 1000000000.;
    }

    void reset() { clock_gettime(CLOCK_REALTIME, &beg_); }

private:
    timespec beg_, end_;
};

Example of usage:

int main()
{
    Timer tmr;
    double t = tmr.elapsed();
    std::cout << t << std::endl;

    tmr.reset();
    t = tmr.elapsed();
    std::cout << t << std::endl;
    return 0;
}

From https://gist.github.com/gongzhitaao/7062087

Really killing a process in Windows

Process Hacker has numerous ways of killing a process.

(Right-click the process, then go to Miscellaneous->Terminator.)

Appending to an object

How about storing the alerts as records in an array instead of properties of a single object ?

var alerts = [ 
    {num : 1, app:'helloworld',message:'message'},
    {num : 2, app:'helloagain',message:'another message'} 
]

And then to add one, just use push:

alerts.push({num : 3, app:'helloagain_again',message:'yet another message'});

DataGridView AutoFit and Fill

This is my favorite approach...

_dataGrid.DataBindingComplete += (o, _) =>
    {
        var dataGridView = o as DataGridView;
        if (dataGridView != null)
        {
           dataGridView.AutoSizeColumnsMode = DataGridViewAutoSizeColumnsMode.AllCells;
           dataGridView.Columns[dataGridView.ColumnCount-1].AutoSizeMode = DataGridViewAutoSizeColumnMode.Fill;
        }
    };

How do I restrict a float value to only two places after the decimal point in C?

Assuming you're talking about round the value for printing, then Andrew Coleson and AraK's answer are correct:

printf("%.2f", 37.777779);

But note that if you're aiming to round the number to exactly 37.78 for internal use (eg to compare against another value), then this isn't a good idea, due to the way floating point numbers work: you usually don't want to do equality comparisons for floating point, instead use a target value +/- a sigma value. Or encode the number as a string with a known precision, and compare that.

See the link in Greg Hewgill's answer to a related question, which also covers why you shouldn't use floating point for financial calculations.

Shuffle DataFrame rows

(I don't have enough reputation to comment this on the top post, so I hope someone else can do that for me.) There was a concern raised that the first method:

df.sample(frac=1)

made a deep copy or just changed the dataframe. I ran the following code:

print(hex(id(df)))
print(hex(id(df.sample(frac=1))))
print(hex(id(df.sample(frac=1).reset_index(drop=True))))

and my results were:

0x1f8a784d400
0x1f8b9d65e10
0x1f8b9d65b70

which means the method is not returning the same object, as was suggested in the last comment. So this method does indeed make a shuffled copy.

How to set the min and max height or width of a Frame?

A workaround - at least for the minimum size: You can use grid to manage the frames contained in root and make them follow the grid size by setting sticky='nsew'. Then you can use root.grid_rowconfigure and root.grid_columnconfigure to set values for minsize like so:

from tkinter import Frame, Tk

class MyApp():
    def __init__(self):
        self.root = Tk()

        self.my_frame_red = Frame(self.root, bg='red')
        self.my_frame_red.grid(row=0, column=0, sticky='nsew')

        self.my_frame_blue = Frame(self.root, bg='blue')
        self.my_frame_blue.grid(row=0, column=1, sticky='nsew')

        self.root.grid_rowconfigure(0, minsize=200, weight=1)
        self.root.grid_columnconfigure(0, minsize=200, weight=1)
        self.root.grid_columnconfigure(1, weight=1)

        self.root.mainloop()

if __name__ == '__main__':
    app = MyApp()

But as Brian wrote (in 2010 :D) you can still resize the window to be smaller than the frame if you don't limit its minsize.

Git Server Like GitHub?

for simple setups, you can provide ssh access to a central server, and set up each user's working directory to push/pull from this 'origin'. That would be the most simple and common setup for small teams.

You might also look into gitosis which gives you an http server and the ability to manage it remotely. That way you don't have to grant ssh access and all that entails to each committer.

How to test android apps in a real device with Android Studio?

For Android 7, Galaxy S6 Edge:

  1. Settings > Developer Options > Turn the switch ON > Debugging Mode (Turn On)

If Developer Options is not available then

  1. Settings > About Device > Software Info > Build number (Tap It 7 time)

Now perform step 1. Now it should work, if its still not working then perform these steps. It worked for me.

Differences between Microsoft .NET 4.0 full Framework and Client Profile

You should deploy "Client Profile" instead of "Full Framework" inside a corporation mostly in one case only: you want explicitly deny some .NET features are running on the client computers. The only real case is denying of ASP.NET on the client machines of the corporation, for example, because of security reasons or the existing corporate policy.

Saving of less than 8 MB on client computer can not be a serious reason of "Client Profile" deployment in a corporation. The risk of the necessity of the deployment of the "Full Framework" later in the corporation is higher than costs of 8 MB per client.

receiver type *** for instance message is a forward declaration

I was trying to use @class "Myclass.h".

When I changed it to #import "Myclass.h", it worked fine.

Trying to check if username already exists in MySQL database using PHP

$query = mysql_query("SELECT username FROM Users WHERE username='$username' ")

Use prepared statements, do not use mysql as it is deprecated.

// check if name is taken already
$stmt = $link->prepare("SELECT username FROM users WHERE username = :username");
$stmt->execute([
    'username' => $username
]);
$user = $stmt->fetch(PDO::FETCH_ASSOC);

if (isset($user) && !empty($user)){
    // Username already taken
}

Converting LastLogon to DateTime format

DateTime.FromFileTime should do the trick:

PS C:\> [datetime]::FromFileTime(129948127853609000)

Monday, October 15, 2012 3:13:05 PM

Then depending on how you want to format it, check out standard and custom datetime format strings.

PS C:\> [datetime]::FromFileTime(129948127853609000).ToString('d MMMM')
15 October
PS C:\> [datetime]::FromFileTime(129948127853609000).ToString('g')
10/15/2012 3:13 PM

If you want to integrate this into your one-liner, change your select statement to this:

... | Select Name, manager, @{N='LastLogon'; E={[DateTime]::FromFileTime($_.LastLogon)}} | ...

What is exactly the base pointer and stack pointer? To what do they point?

EDIT: For a better description, see x86 Disassembly/Functions and Stack Frames in a WikiBook about x86 assembly. I try to add some info you might be interested in using Visual Studio.

Storing the caller EBP as the first local variable is called a standard stack frame, and this may be used for nearly all calling conventions on Windows. Differences exist whether the caller or callee deallocates the passed parameters, and which parameters are passed in registers, but these are orthogonal to the standard stack frame problem.

Speaking about Windows programs, you might probably use Visual Studio to compile your C++ code. Be aware that Microsoft uses an optimization called Frame Pointer Omission, that makes it nearly impossible to do walk the stack without using the dbghlp library and the PDB file for the executable.

This Frame Pointer Omission means that the compiler does not store the old EBP on a standard place and uses the EBP register for something else, therefore you have hard time finding the caller EIP without knowing how much space the local variables need for a given function. Of course Microsoft provides an API that allows you to do stack-walks even in this case, but looking up the symbol table database in PDB files takes too long for some use cases.

To avoid FPO in your compilation units, you need to avoid using /O2 or need to explicitly add /Oy- to the C++ compilation flags in your projects. You probably link against the C or C++ runtime, which uses FPO in the Release configuration, so you will have hard time to do stack walks without the dbghlp.dll.

"SyntaxError: Unexpected token < in JSON at position 0"

Those who are using create-react-app and trying to fetch local json files.

As in create-react-app, webpack-dev-server is used to handle the request and for every request it serves the index.html. So you are getting

SyntaxError: Unexpected token < in JSON at position 0.

To solve this, you need to eject the app and modify the webpack-dev-server configuration file.

You can follow the steps from here.

jQuery select all except first

My answer is focused to a extended case derived from the one exposed at top.

Suppose you have group of elements from which you want to hide the child elements except first. As an example:

<html>
  <div class='some-group'>
     <div class='child child-0'>visible#1</div>
     <div class='child child-1'>xx</div>
     <div class='child child-2'>yy</div>
  </div>
  <div class='some-group'>
     <div class='child child-0'>visible#2</div>
     <div class='child child-1'>aa</div>
     <div class='child child-2'>bb</div>
  </div>
</html>
  1. We want to hide all .child elements on every group. So this will not help because will hide all .child elements except visible#1:

    $('.child:not(:first)').hide();
    
  2. The solution (in this extended case) will be:

    $('.some-group').each(function(i,group){
        $(group).find('.child:not(:first)').hide();
    });
    

How do I get the computer name in .NET

Try this one.

public static string GetFQDN()
{
    string domainName = NetworkInformation.IPGlobalProperties.GetIPGlobalProperties().DomainName;
    string hostName = Dns.GetHostName();
    string fqdn;
    if (!hostName.Contains(domainName))
        fqdn = hostName + "." +domainName;
    else
        fqdn = hostName;

    return fqdn;
}

What is the MySQL VARCHAR max size?

Source

The max length of a varchar is subject to the max row size in MySQL, which is 64KB (not counting BLOBs):

VARCHAR(65535) However, note that the limit is lower if you use a multi-byte character set:

VARCHAR(21844) CHARACTER SET utf8

Clicking a checkbox with ng-click does not update the model

Usually this is due to another directive in-between your ng-controller and your input that is creating a new scope. When the select writes out it value, it will write it up to the most recent scope, so it would write it to this scope rather than the parent that is further away.

The best practice is to never bind directly to a variable on the scope in an ng-model, this is also known as always including a "dot" in your ngmodel. For a better explanation of this, check out this video from John:

http://www.youtube.com/watch?v=DTx23w4z6Kc

Solution from: https://groups.google.com/forum/#!topic/angular/7Nd_me5YrHU

How to use MapView in android using google map V2?

yes you can use MapView in v2... for further details you can get help from this

https://gist.github.com/joshdholtz/4522551


SomeFragment.java

public class SomeFragment extends Fragment implements OnMapReadyCallback{
 
    MapView mapView;
    GoogleMap map;
 
    @Override
    public View onCreateView(LayoutInflater inflater, ViewGroup container, Bundle savedInstanceState) {
        View v = inflater.inflate(R.layout.some_layout, container, false);
 
        // Gets the MapView from the XML layout and creates it
        mapView = (MapView) v.findViewById(R.id.mapview);
        mapView.onCreate(savedInstanceState);
 
    
        mapView.getMapAsync(this);
        
 
        return v;
    }
 
   @Override
   public void onMapReady(GoogleMap googleMap) {
       map = googleMap;
       map.getUiSettings().setMyLocationButtonEnabled(false);
       map.setMyLocationEnabled(true);
       /*
       //in old Api Needs to call MapsInitializer before doing any CameraUpdateFactory call
        try {
            MapsInitializer.initialize(this.getActivity());
        } catch (GooglePlayServicesNotAvailableException e) {
            e.printStackTrace();
        } 
       */
        
        // Updates the location and zoom of the MapView
        /*CameraUpdate cameraUpdate = CameraUpdateFactory.newLatLngZoom(new LatLng(43.1, -87.9), 10);
        map.animateCamera(cameraUpdate);*/
        map.moveCamera(CameraUpdateFactory.newLatLng(new LatLng(43.1, -87.9)));

    }

    @Override
    public void onResume() {
        mapView.onResume();
        super.onResume();
    }


    @Override
    public void onPause() {
        super.onPause();
        mapView.onPause();
    }
 
    @Override
    public void onDestroy() {
        super.onDestroy();
        mapView.onDestroy();
    }
 
    @Override
    public void onLowMemory() {
        super.onLowMemory();
        mapView.onLowMemory();
    }
 
}

AndroidManifest.xml

<manifest xmlns:android="http://schemas.android.com/apk/res/android"
    package="com.example"
    android:versionCode="1"
    android:versionName="1.0" >
    
    <uses-sdk
        android:minSdkVersion="8"
        android:targetSdkVersion="15" />
    
    <uses-permission android:name="android.permission.INTERNET"/>
    <uses-permission android:name="com.google.android.providers.gsf.permission.READ_GSERVICES"/>
    <uses-permission android:name="android.permission.ACCESS_COARSE_LOCATION"/>
    <uses-permission android:name="android.permission.ACCESS_FINE_LOCATION"/>
    
    <uses-feature
        android:glEsVersion="0x00020000"
        android:required="true"/>
    
    <permission
        android:name="com.example.permission.MAPS_RECEIVE"
        android:protectionLevel="signature"/>
    <uses-permission android:name="com.example.permission.MAPS_RECEIVE"/>
    
    <application
        android:icon="@drawable/ic_launcher"
        android:label="@string/app_name"
        android:theme="@style/AppTheme" >
        
        <meta-data
            android:name="com.google.android.maps.v2.API_KEY"
            android:value="your_key"/>
        
        <activity
            android:name=".HomeActivity"
            android:label="@string/app_name" >
            <intent-filter>
                <action android:name="android.intent.action.MAIN" />
                <category android:name="android.intent.category.LAUNCHER" />
            </intent-filter>
        </activity>
    
    </application>
 
</manifest>

some_layout.xml

<LinearLayout xmlns:android="http://schemas.android.com/apk/res/android"
    xmlns:tools="http://schemas.android.com/tools"
    android:layout_width="fill_parent"
    android:layout_height="fill_parent" >
    
    <com.google.android.gms.maps.MapView android:id="@+id/mapview"
        android:layout_width="fill_parent" 
        android:layout_height="fill_parent" />
 
</LinearLayout>

How to tell if string starts with a number with Python?

Python's string library has isdigit() method:

string[0].isdigit()

Change a column type from Date to DateTime during ROR migration

In Rails 3.2 and Rails 4, Benjamin's popular answer has a slightly different syntax.

First in your terminal:

$ rails g migration change_date_format_in_my_table

Then in your migration file:

class ChangeDateFormatInMyTable < ActiveRecord::Migration
  def up
   change_column :my_table, :my_column, :datetime
  end

  def down
   change_column :my_table, :my_column, :date
  end
end

T-SQL Cast versus Convert

CAST uses ANSI standard. In case of portability, this will work on other platforms. CONVERT is specific to sql server. But is very strong function. You can specify different styles for dates

Why does cURL return error "(23) Failed writing body"?

I encountered the same problem when doing:

curl -L https://packagecloud.io/golang-migrate/migrate/gpgkey | apt-key add -

The above query needs to be executed using root privileges.

Writing it in following way solved the issue for me:

curl -L https://packagecloud.io/golang-migrate/migrate/gpgkey | sudo apt-key add -

If you write sudo before curl, you will get the Failed writing body error.

Window.open and pass parameters by post method

Even though I am 3 years late, but to simplify Guffa's example, you don't even need to have the form on the page at all:

$('<form method="post" action="test.asp" target="TheWindow">
       <input type="hidden" name="something" value="something">
       ...
   </form>').submit();

Edited:

$('<form method="post" action="test.asp" target="TheWindow">
       <input type="hidden" name="something" value="something">
       ...
   </form>').appendTo('body').submit().remove();

Maybe a helpful tip for someone :)

Removing element from array in component state

I want to chime in here even though this question has already been answered correctly by @pscl in case anyone else runs into the same issue I did. Out of the 4 methods give I chose to use the es6 syntax with arrow functions due to it's conciseness and lack of dependence on external libraries:

Using Array.prototype.filter with ES6 Arrow Functions

removeItem(index) {
  this.setState((prevState) => ({
    data: prevState.data.filter((_, i) => i != index)
  }));
}

As you can see I made a slight modification to ignore the type of index (!== to !=) because in my case I was retrieving the index from a string field.

Another helpful point if you're seeing weird behavior when removing an element on the client side is to NEVER use the index of an array as the key for the element:

// bad
{content.map((content, index) =>
  <p key={index}>{content.Content}</p>
)}

When React diffs with the virtual DOM on a change, it will look at the keys to determine what has changed. So if you're using indices and there is one less in the array, it will remove the last one. Instead, use the id's of the content as keys, like this.

// good
{content.map(content =>
  <p key={content.id}>{content.Content}</p>
)}

The above is an excerpt from this answer from a related post.

Happy Coding Everyone!

Stop fixed position at footer

I've made some changes to the second most popular answer as i found this worked better for me. The changes use window.innerHeight as it is more dynamic than adding your own height for the nav (above used + 570). this allows the code to work on mobile, tablet and desktop dynamicly.

$(window).scroll(() => {
            //Distance from top fo document to top of footer
            topOfFooter = $('#footer').position().top;
             // Distance user has scrolled from top + windows inner height
             scrollDistanceFromTopOfDoc = $(document).scrollTop() + window.innerHeight;
             // Difference between the two.
             scrollDistanceFromTopOfFooter = scrollDistanceFromTopOfDoc - topOfFooter; 
            // If user has scrolled further than footer,
              if (scrollDistanceFromTopOfDoc > topOfFooter) {
                // add margin-bottom so button stays above footer.
                $('#floating-button').css('margin-bottom',  0 + scrollDistanceFromTopOfFooter);
              } else  {
                // remove margin-bottom so button goes back to the bottom of the page
                $('#floating-button').css('margin-bottom', 0);
              }
            });

RabbitMQ / AMQP: single queue, multiple consumers for same message?

Just read the rabbitmq tutorial. You publish message to exchange, not to queue; it is then routed to appropriate queues. In your case, you should bind separate queue for each consumer. That way, they can consume messages completely independently.

GitHub: Permission denied (publickey). fatal: The remote end hung up unexpectedly

I tried the solutions mentioned but still failed. I found the solution that finally worked for me here - removing then re-adding the remote link

Why are there no ++ and --? operators in Python?

The ++ class of operators are expressions with side effects. This is something generally not found in Python.

For the same reason an assignment is not an expression in Python, thus preventing the common if (a = f(...)) { /* using a here */ } idiom.

Lastly I suspect that there operator are not very consistent with Pythons reference semantics. Remember, Python does not have variables (or pointers) with the semantics known from C/C++.

How to yum install Node.JS on Amazon Linux

As others mentioned using epel gives a really outdated version, here is a little script I just wrote instead to add to the CI pipeline or pass it to ec2 user-data to install the latest version of node, simply replace the version with what you want, and the appropriate distro of Linux you are using.

The following example is for amazon-Linux-2-AMI

#!/bin/bash

version='v14.13.1'
distro='linux-x64'
package_name="node-$version-$distro"
package_location="/usr/local/lib/"

curl -O https://nodejs.org/download/release/latest/$package_name.tar.gz
tar -xvf $package_name.tar.gz -C $package_location
rm -rfv $package_name.tar.gz

echo "export PATH=$package_location/$package_name/bin:\$PATH" >> ~/.profile

if you want to test it in the same shell simply run

. ~/.profile

Can I get the name of the currently running function in JavaScript?

Since you have written a function named foo and you know it is in myfile.js why do you need to get this information dynamically?

That being said you can use arguments.callee.toString() inside the function (this is a string representation of the entire function) and regex out the value of the function name.

Here is a function that will spit out its own name:

function foo() {
    re = /^function\s+([^(]+)/
    alert(re.exec(arguments.callee.toString())[1]);             
}

jQuery AJAX cross domain

From the Jquery docs (link):

  • Due to browser security restrictions, most "Ajax" requests are subject to the same origin policy; the request can not successfully retrieve data from a different domain, subdomain, or protocol.

  • Script and JSONP requests are not subject to the same origin policy restrictions.

So I would take it that you need to use jsonp for the request. But haven't tried this myself.

Putting an if-elif-else statement on one line?

You can optionally actually use the get method of a dict:

x = {i<100: -1, -10<=i<=10: 0, i>100: 1}.get(True, 2)

You don't need the get method if one of the keys is guaranteed to evaluate to True:

x = {i<0: -1, i==0: 0, i>0: 1}[True]

At most one of the keys should ideally evaluate to True. If more than one key evaluates to True, the results could seem unpredictable.

Solr vs. ElasticSearch

I see some of the above answers are now a bit out of date. From my perspective, and I work with both Solr(Cloud and non-Cloud) and ElasticSearch on a daily basis, here are some interesting differences:

  • Community: Solr has a bigger, more mature user, dev, and contributor community. ES has a smaller, but active community of users and a growing community of contributors
  • Maturity: Solr is more mature, but ES has grown rapidly and I consider it stable
  • Performance: hard to judge. I/we have not done direct performance benchmarks. A person at LinkedIn did compare Solr vs. ES vs. Sensei once, but the initial results should be ignored because they used non-expert setup for both Solr and ES.
  • Design: People love Solr. The Java API is somewhat verbose, but people like how it's put together. Solr code is unfortunately not always very pretty. Also, ES has sharding, real-time replication, document and routing built-in. While some of this exists in Solr, too, it feels a bit like an after-thought.
  • Support: there are companies providing tech and consulting support for both Solr and ElasticSearch. I think the only company that provides support for both is Sematext (disclosure: I'm Sematext founder)
  • Scalability: both can be scaled to very large clusters. ES is easier to scale than pre-Solr 4.0 version of Solr, but with Solr 4.0 that's no longer the case.

For more thorough coverage of Solr vs. ElasticSearch topic have a look at https://sematext.com/blog/solr-vs-elasticsearch-part-1-overview/ . This is the first post in the series of posts from Sematext doing direct and neutral Solr vs. ElasticSearch comparison. Disclosure: I work at Sematext.

How to compare character ignoring case in primitive types

This is how the JDK does it (adapted from OpenJDK 8, String.java/regionMatches):

static boolean charactersEqualIgnoringCase(char c1, char c2) {
  if (c1 == c2) return true;

  // If characters don't match but case may be ignored,
  // try converting both characters to uppercase.
  char u1 = Character.toUpperCase(c1);
  char u2 = Character.toUpperCase(c2);
  if (u1 == u2) return true;

  // Unfortunately, conversion to uppercase does not work properly
  // for the Georgian alphabet, which has strange rules about case
  // conversion.  So we need to make one last check before
  // exiting.
  return Character.toLowerCase(u1) == Character.toLowerCase(u2);
}

I suppose that works for Turkish too.

How to create a session using JavaScript?

You can use Local storage.

local storage is same as session. the data will be removed when you close the browser.

<script> 
localStorage.setItem('lastname','Smith');

alert(localStorage.getItem('lastname'));
</script>

Edit: @Naveen comment below is correct. You can use session storage instead. (https://www.w3schools.com/jsref/prop_win_sessionstorage.asp) Thanks.

<script> 
sessionStorage.setItem('lastname','Smith');

alert(sessionStorage.getItem('lastname'));
</script>

How to enable authentication on MongoDB through Docker?

a. You can use environment variables via terminal:

$ docker run -d --name container_name \
      -e MONGO_INITDB_ROOT_USERNAME=admin \
      -e MONGO_INITDB_ROOT_PASSWORD=password \
      mongo

If you like to test if everything works:

// ssh into the running container
// Change container name if necessary
$ docker exec -it mongo /bin/bash

// Enter into mongo shell
$ mongo

// Caret will change when you enter successfully
// Switch to admin database
$> use admin
$> db.auth("admin", passwordPrompt())

// Show available databases
$> show dbs

If you like to instantiate a database on first run, check option b.

b. You can use environment variables in your docker stack deploy file or compose file for versions 3.4 through 4.1.

As it is explained on the quick reference section of the official mongo image set MONGO_INITDB_ROOT_USERNAME and MONGO_INITDB_ROOT_PASSWORD in your yaml file:

mongo:
    image: mongo
    environment:
      MONGO_INITDB_ROOT_USERNAME: admin
      MONGO_INITDB_ROOT_PASSWORD: password

docker-entrypoint.sh file in mongo image checks for the existence of these two variables and sets --auth flag accordingly.

c. You can also use docker secrets.

MONGO_INITDB_ROOT_USERNAME and MONGO_INITDB_ROOT_PASSWORD is set indirectly by docker-entrypoint.sh from MONGO_INITDB_ROOT_USERNAME_FILE and MONGO_INITDB_ROOT_PASSWORD_FILE variables:

mongo:
    image: mongo
    environment:
        - MONGO_INITDB_ROOT_USERNAME_FILE=/run/secrets/db_root_username
        - MONGO_INITDB_ROOT_PASSWORD_FILE=/run/secrets/db_root_password
    secrets:
      - db_root_username
      - db_root_password

docker-entrypoint.sh converts MONGO_INITDB_ROOT_USERNAME_FILE and MONGO_INITDB_ROOT_PASSWORD_FILE to MONGO_INITDB_ROOT_USERNAME and MONGO_INITDB_ROOT_PASSWORD.

You can use MONGO_INITDB_ROOT_USERNAME and MONGO_INITDB_ROOT_PASSWORD in your .sh or .js scripts in docker-entrypoint-initdb.d folder while initializing database instance.

When a container is started for the first time it will execute files with extensions .sh and .js that are found in /docker-entrypoint-initdb.d. Files will be executed in alphabetical order. .js files will be executed by mongo using the database specified by the MONGO_INITDB_DATABASE variable, if it is present, or test otherwise. You may also switch databases within the .js script.

This last method is not in the reference docs, so it may not survive an update.

Automatically create requirements.txt

if you are using PyCharm, when you open or clone the project into the PyCharm it shows an alert and ask you for installing all necessary packages.

Eclipse: Java was started but returned error code=13

Since you didn't mention the version of Eclipse, I advice you to download the latest version of Eclipse Luna which comes with Java 8 support by default.

Can't find AVD or SDK manager in Eclipse

I have solved this as follows:

  1. Window > Customize Perspective... (you will see Android and AVD Manager are disabled)

  2. Command Groups Availability > Android and AVD Manager > check

  3. Tool Bar Visibility > Android and AVD Manager > check

Current time formatting with Javascript

A JavaScript Date has several methods allowing you to extract its parts:

getFullYear() - Returns the 4-digit year
getMonth() - Returns a zero-based integer (0-11) representing the month of the year.
getDate() - Returns the day of the month (1-31).
getDay() - Returns the day of the week (0-6). 0 is Sunday, 6 is Saturday.
getHours() - Returns the hour of the day (0-23).
getMinutes() - Returns the minute (0-59).
getSeconds() - Returns the second (0-59).
getMilliseconds() - Returns the milliseconds (0-999).
getTimezoneOffset() - Returns the number of minutes between the machine local time and UTC.

There are no built-in methods allowing you to get localized strings like "Friday", "February", or "PM". You have to code that yourself. To get the string you want, you at least need to store string representations of days and months:

var months = ["Jan", "Feb", "Mar", "Apr", "May", "Jun", "Jul", "Aug", "Sep", "Oct", "Nov", "Dec"];
var days = ["Sunday", "Monday", "Tuesday", "Wednesday", "Thursday", "Friday", "Saturday"];

Then, put it together using the methods above:

_x000D_
_x000D_
var months = ["Jan", "Feb", "Mar", "Apr", "May", "Jun", "Jul", "Aug", "Sep", "Oct", "Nov", "Dec"];_x000D_
var days = ["Sunday", "Monday", "Tuesday", "Wednesday", "Thursday", "Friday", "Saturday"];_x000D_
var d = new Date();_x000D_
var day = days[d.getDay()];_x000D_
var hr = d.getHours();_x000D_
var min = d.getMinutes();_x000D_
if (min < 10) {_x000D_
    min = "0" + min;_x000D_
}_x000D_
var ampm = "am";_x000D_
if( hr > 12 ) {_x000D_
    hr -= 12;_x000D_
    ampm = "pm";_x000D_
}_x000D_
var date = d.getDate();_x000D_
var month = months[d.getMonth()];_x000D_
var year = d.getFullYear();_x000D_
var x = document.getElementById("time");_x000D_
x.innerHTML = day + " " + hr + ":" + min + ampm + " " + date + " " + month + " " + year;
_x000D_
<span id="time"></span>
_x000D_
_x000D_
_x000D_

I have a date format function I like to include in my standard library. It takes a format string parameter that defines the desired output. The format strings are loosely based on .Net custom Date and Time format strings. For the format you specified the following format string would work: "dddd h:mmtt d MMM yyyy".

var d = new Date();
var x = document.getElementById("time");
x.innerHTML = formatDate(d, "dddd h:mmtt d MMM yyyy");

Demo: jsfiddle.net/BNkkB/1

Here is my full date formatting function:

function formatDate(date, format, utc) {
    var MMMM = ["\x00", "January", "February", "March", "April", "May", "June", "July", "August", "September", "October", "November", "December"];
    var MMM = ["\x01", "Jan", "Feb", "Mar", "Apr", "May", "Jun", "Jul", "Aug", "Sep", "Oct", "Nov", "Dec"];
    var dddd = ["\x02", "Sunday", "Monday", "Tuesday", "Wednesday", "Thursday", "Friday", "Saturday"];
    var ddd = ["\x03", "Sun", "Mon", "Tue", "Wed", "Thu", "Fri", "Sat"];

    function ii(i, len) {
        var s = i + "";
        len = len || 2;
        while (s.length < len) s = "0" + s;
        return s;
    }

    var y = utc ? date.getUTCFullYear() : date.getFullYear();
    format = format.replace(/(^|[^\\])yyyy+/g, "$1" + y);
    format = format.replace(/(^|[^\\])yy/g, "$1" + y.toString().substr(2, 2));
    format = format.replace(/(^|[^\\])y/g, "$1" + y);

    var M = (utc ? date.getUTCMonth() : date.getMonth()) + 1;
    format = format.replace(/(^|[^\\])MMMM+/g, "$1" + MMMM[0]);
    format = format.replace(/(^|[^\\])MMM/g, "$1" + MMM[0]);
    format = format.replace(/(^|[^\\])MM/g, "$1" + ii(M));
    format = format.replace(/(^|[^\\])M/g, "$1" + M);

    var d = utc ? date.getUTCDate() : date.getDate();
    format = format.replace(/(^|[^\\])dddd+/g, "$1" + dddd[0]);
    format = format.replace(/(^|[^\\])ddd/g, "$1" + ddd[0]);
    format = format.replace(/(^|[^\\])dd/g, "$1" + ii(d));
    format = format.replace(/(^|[^\\])d/g, "$1" + d);

    var H = utc ? date.getUTCHours() : date.getHours();
    format = format.replace(/(^|[^\\])HH+/g, "$1" + ii(H));
    format = format.replace(/(^|[^\\])H/g, "$1" + H);

    var h = H > 12 ? H - 12 : H == 0 ? 12 : H;
    format = format.replace(/(^|[^\\])hh+/g, "$1" + ii(h));
    format = format.replace(/(^|[^\\])h/g, "$1" + h);

    var m = utc ? date.getUTCMinutes() : date.getMinutes();
    format = format.replace(/(^|[^\\])mm+/g, "$1" + ii(m));
    format = format.replace(/(^|[^\\])m/g, "$1" + m);

    var s = utc ? date.getUTCSeconds() : date.getSeconds();
    format = format.replace(/(^|[^\\])ss+/g, "$1" + ii(s));
    format = format.replace(/(^|[^\\])s/g, "$1" + s);

    var f = utc ? date.getUTCMilliseconds() : date.getMilliseconds();
    format = format.replace(/(^|[^\\])fff+/g, "$1" + ii(f, 3));
    f = Math.round(f / 10);
    format = format.replace(/(^|[^\\])ff/g, "$1" + ii(f));
    f = Math.round(f / 10);
    format = format.replace(/(^|[^\\])f/g, "$1" + f);

    var T = H < 12 ? "AM" : "PM";
    format = format.replace(/(^|[^\\])TT+/g, "$1" + T);
    format = format.replace(/(^|[^\\])T/g, "$1" + T.charAt(0));

    var t = T.toLowerCase();
    format = format.replace(/(^|[^\\])tt+/g, "$1" + t);
    format = format.replace(/(^|[^\\])t/g, "$1" + t.charAt(0));

    var tz = -date.getTimezoneOffset();
    var K = utc || !tz ? "Z" : tz > 0 ? "+" : "-";
    if (!utc) {
        tz = Math.abs(tz);
        var tzHrs = Math.floor(tz / 60);
        var tzMin = tz % 60;
        K += ii(tzHrs) + ":" + ii(tzMin);
    }
    format = format.replace(/(^|[^\\])K/g, "$1" + K);

    var day = (utc ? date.getUTCDay() : date.getDay()) + 1;
    format = format.replace(new RegExp(dddd[0], "g"), dddd[day]);
    format = format.replace(new RegExp(ddd[0], "g"), ddd[day]);

    format = format.replace(new RegExp(MMMM[0], "g"), MMMM[M]);
    format = format.replace(new RegExp(MMM[0], "g"), MMM[M]);

    format = format.replace(/\\(.)/g, "$1");

    return format;
};

ojdbc14.jar vs. ojdbc6.jar

I have same problem!

Found following in oracle site link text

As mentioned above, the 11.1 drivers by default convert SQL DATE to Timestamp when reading from the database. This always was the right thing to do and the change in 9i was a mistake. The 11.1 drivers have reverted to the correct behavior. Even if you didn't set V8Compatible in your application you shouldn't see any difference in behavior in most cases. You may notice a difference if you use getObject to read a DATE column. The result will be a Timestamp rather than a Date. Since Timestamp is a subclass of Date this generally isn't a problem. Where you might notice a difference is if you relied on the conversion from DATE to Date to truncate the time component or if you do toString on the value. Otherwise the change should be transparent.

If for some reason your app is very sensitive to this change and you simply must have the 9i-10g behavior, there is a connection property you can set. Set mapDateToTimestamp to false and the driver will revert to the default 9i-10g behavior and map DATE to Date.

Break when a value changes using the Visual Studio debugger

Update in 2019:

This is now officially supported in Visual Studio 2019 Preview 2 for .Net Core 3.0 or higher. Of course, you may have to put some thoughts in potential risks of using a Preview version of IDE. I imagine in the near future this will be included in the official Visual Studio.

https://blogs.msdn.microsoft.com/visualstudio/2019/02/12/break-when-value-changes-data-breakpoints-for-net-core-in-visual-studio-2019/

Fortunately, data breakpoints are no longer a C++ exclusive because they are now available for .NET Core (3.0 or higher) in Visual Studio 2019 Preview 2!

login to remote using "mstsc /admin" with password

the command posted by Milad and Sandy did not work for me with mstsc. i had to add TERMSRV to the /generic switch. i found this information here: https://gist.github.com/jdforsythe/48a022ee22c8ec912b7e

cmdkey /generic:TERMSRV/<server> /user:<username> /pass:<password>

i could then use mstsc /v:<server> without getting prompted for the login.

Rename Oracle Table or View

One can rename indexes the same way:

alter index owner.index_name rename to new_name;

Angular directives - when and how to use compile, controller, pre-link and post-link

What is the difference between a source template and an instance template?

The fact that Angular allows DOM manipulation means that the input markup into the compilation process sometimes differ from the output. Particularly, some input markup may be cloned a few times (like with ng-repeat) before being rendered to the DOM.

Angular terminology is a bit inconsistent, but it still distinguishes between two types of markups:

  • Source template - the markup to be cloned, if needed. If cloned, this markup will not be rendered to the DOM.
  • Instance template - the actual markup to be rendered to the DOM. If cloning is involved, each instance will be a clone.

The following markup demonstrates this:

<div ng-repeat="i in [0,1,2]">
    <my-directive>{{i}}</my-directive>
</div>

The source html defines

    <my-directive>{{i}}</my-directive>

which serves as the source template.

But as it is wrapped within an ng-repeat directive, this source template will be cloned (3 times in our case). These clones are instance template, each will appear in the DOM and be bound to the relevant scope.

SQL Server Management Studio, how to get execution time down to milliseconds

Turn on Client Statistics by doing one of the following:

  • Menu: Query > Include client Statistics
  • Toolbar: Click the button (next to Include Actual Execution Time)
  • Keyboard: Shift-Alt-S

Then you get a new tab which records the timings, IO data and rowcounts etc for (up to) the last 10 exections (plus averages!):

enter image description here

Could not load file or assembly '' or one of its dependencies

I had this today, and in my case the issue was very odd:

  <dependentAssembly>
    <assemblyIdentity name="Microsoft.Owin.Host.SystemWeb" publicKeyToken="31bf3856ad364e35" culture="neutral" />
    <bindingRedirect oldVersion="0.0.0.0-3.1.0" newVersion="3.1.0.0" />
  </dependentAssembly>0.

Note the stray characters at the end of the XML - somehow those had been moved from the version number to the end of this block of XML!

  <dependentAssembly>
    <assemblyIdentity name="Microsoft.Owin.Host.SystemWeb" publicKeyToken="31bf3856ad364e35" culture="neutral" />
    <bindingRedirect oldVersion="0.0.0.0-3.1.0.0" newVersion="3.1.0.0" />
  </dependentAssembly>

Changed to the above and voila! Everything worked again.